Categories
Exam Questions M.I.T.

M.I.T. Midterm and final exam questions for first half of international economics. Kindleberger, 1961-1967

 

The two term graduate sequence for international economics 14.581 and 14.582 provided the following course description in the M.I.T. catalogues, unchanged over the better part of the 1950’s and 1960’s:

The foreign exchange market, foreign trade and commercial policy, with emphasis on the relation of the items in the current account to national income, international finance and the achievement and maintenance of equibrium in the balance of payments as a whole; current problems of international economics.

For this post I have transcribed six sets of the 1960’s exams for the first course of the sequence taught by Charles Kindleberger. 

Kindleberger’s exams for both 14.581 and 14.582 for 1954-55 have been posted earlier, as have his exams for 1950-51.

_____________________________

Fall Term 1961-62

14.581 International Economics. Professor C. P. Kindleberger.  3 hours/week, 37 Students.

 

14.581
November 9, 1961
HOUR QUIZ

Answer two questions (equal weight).

  1. Discus some of the choices which balance-of-payments statisticians must make, and illustrate how the outcomes are governed by the purposes to be served on the one hand, and the nature of the raw material on the other.
  2. Indicate the contribution which the establishment of a forward market can make to hedging facilities for foreign traders
  3. Evaluate the Heckscher-Ohlin theorem as an explanation of comparative advantage.

 

14.581 – International Economics
FINAL EXAMINATION
C. P. Kindleberger
January 23, 1962

NO BOOKS ALLOWED.
Answer question 1 and any three of the following five.

  1. (one hour) Discuss the relevance to the theory of international trade taken in the widest sense of any three of the classical assumptions of:

a) full employment
b) mobility of resources within but not between countries
c) perfect competition
d) the labor theory of value
e) Say’s Law of markets

How is the theory modified, and the prescription of free trade altered, if the assumptions you deal with have to be revised?

Answer three questions (forty minutes each).

  1. Which side do you favor in the debate between the elasticities and absorption in the exchange -devaluation problem? Explain.
  2. To what extent, if at all, does international trade theory illuminate the tariff history of some country with which you are familiar? Give details.
  3. How do tariffs affect the distribution of income within and between countries? Illustrate, with reference to the relevant theorems.
  4. Under what circumstances, if ever, are two of the following three weapons of commercial policy justified: a) tariffs; b) quota restrictions; c) foreign exchange control? Compare the measures you treat with alternative means of achieving the same goals, and include in your justification, if you find one, reasons for why the means indicated are superior to the alternatives.
  5. How is the theory of international trade, and of commercial policy, altered by moving from two to a greater number of countries?

_____________________________

Fall Term 1962-63

14.581 International Economics. Professor C. P. Kindleberger. 3 Hours/week, 46 Students.

Quiz
14.581
November 6, 1962

Answer both questions. (25 minutes each)

  1. How does the United States Department of Commerce define a “deficit” in the balance of payments? Comment on the adequacy of this definition.
  2. Evaluate the success of the Heckscher-Ohlin theory in explaining the basis of international trade.

 

 

Tuesday, January 22, 1963
Time 1:30 – 4:30 P.M.

MASSACHUSETTS INSTITUTE OF TECHNOLOGY
Scheduled Examination in
INTERNATIONAL ECONOMICS 14.581

NOTE: Students are not permitted to use any books, notebooks, or papers in this examination. If brought into the room, they must not be left on the desks

Answer any five questions (36 minutes each).

  1. What difference does the establishment of a forward-exchange market make to the conduct of international trade and exchange?
  2. The underlying theory of international trade is sometimes called a theory of “comparative costs” and sometimes one of “comparative advantage.” Is there any real distinction between these views? Explain in detail.
  3. Explain how trade and restrictions of trade alter the distribution of income within and between countries.
  4. If you were called upon to judge the Alexander-Machlup debate over the adjustment mechanism under changing exchange rates, which side would you favor and why?
  5. What is the “foreign repercussion” in the adjustment mechanism? How does it operate? Evaluate its significance.
  6. What difference does it make, when a country restricts its international trade by a given amount, whether it uses tariffs or quotas?
  7. Do customs unions enlarge welfare?

_____________________________

Fall Term 1963-64

14.581 International Economics. Professor C. P. Kindleberger. 3 Class Hours/Week, 19 Students.

[Note:  one additional section  of 14.581 was taught by L. Lefeber with 22 students]

14.581
One-hour Test
November 14, 1963

Answer both questions, which have equal weight.

  1. What is meant by a deficit in the balance of payments?
  2. Expound the law of comparative advantage in modern economic terms.

 

Tuesday, January 28, 1964
Time: 1.30 – 4.30 P.M.

MASSACHUSETTS INSTITUTE OF TECHNOLOGY
Scheduled Examination in
INTERNATIONAL ECONOMICS – 14.581

NOTE: Students are not permitted to use any books, notebooks or papers in this examination. If brought into the room they must not be left on the desks.

Answer six (6) questions (one-half hour each).

  1. In balance-of-payments accounting, practice differs or is disputed in connection with the following items, among others. What are the various ways in which a country may treat five of them, and what is the justification for each possible treatment?

i) immigrants’ remittances
ii) payments to own nationals for carriage of imports
iii) foreign aid
iii) reinvested profits of foreign-owned enterprises
iv) new gold production sold abroad
v) short-term U.S. claims of commercial banks on foreigners
vi) prepayments of U. S. government loans to foreign governments,

  1. Provide a geometric demonstration of the effect on the terms of trade of technological change in the export good which economizes the scarce factor. State all necessary assumptions explicitly, making them as neutral as possible.
  2. Does the shift of the analysis of the theory of international trade from two to many countries change the theory? In what respects and to what extent?
  3. Explain how currency devaluation under full employment affects the balance of payments, and the terms of trade
  4. Meade states that the adjustment mechanism in international trade is virtually the same under the gold standard and under flexible exchange rates. How does he justify this assertion? Do you agree or disagree? Explain.
  5. The marginal propensity to spend on home goods out of national income in Country A is 2/3rds, and to spend on imports, 1/6. Country B has similar propensities of 1/2 and 1/4. Country A undertakes new expenditure of 100 divided normally between home and abroad. What amount does B have to change its expenditures to preserve internal balance? What happens to A’s balance of payments?
  6. The Reciprocal Trade Agreement Acts of 1934 and thereafter, and the Trade Expansion Act of 1962 called for reciprocal reductions of trade barriers. Under what circumstances and to what extent is it useful for a single country to reduce its tariffs by itself without matching tariff reductions abroad?
  7. Set out at length and in detail the conditions under which customs unions increase world welfare.

_____________________________

Fall Term 1964-65

14.581 International Economics. Professor C. P. Kindleberger. 3 Class Hours/Week, 29 Students.

HOUR TEST
14.581
November 12, 1964

  1. Define accurately “lags and leads” in the balance of payments, and discuss their significance.
  2. What assumption does the Heckscher-Ohlin theorem make about factor inputs of commodities, and what is the significance of this assumption.

 

Tuesday, January 26, 1965
Time: 9:00 – 12:00 A.M.

MASSACHUSETTS INSTITUTE OF TECHNOLOGY
Scheduled Examination in
INTERNATIONAL ECONOMICS – 14.581

Answer one question from each of Groups I to IV, and the single question in Group V.

Group I

  1. Expound the theory of comparative advantage as simply and clearly as you can.
  2. Does it make a significant difference to the theory of international trade to move from an analysis of two to more than two countries? Explain.
  3. What are the gains from trade? How are they distributed? How does the gain of a single country change in response to a change in supply abroad? demand at home?

Group II

  1. Is the purchasing-power-parity doctrine best described as a) a truism; b) a fallacy; c) a useful operational hypothesis? Explain.
  2. Discuss the similarities and differences between the gold standard and the flexible exchange system.

Group III

  1. Is free trade the best policy?
  2. Analyze the slogan “There is nothing that a tariff can do that a subsidy cannot do better”.
  3. Argue for or against international commodity agreements.

Group IV

  1. Does a flexible exchange rate make it possible to pursue an independent monetary and fiscal policy internally? Explain.
  2. What happens to the terms of trade when exchange rates alter?

Group V

  1. What is the effect on its balance of payments of an increase in foreign demand for a country’s exports.

_____________________________

Fall Term 1965-66

14.581 International Economics. Professor C. P. Kindleberger. 3 Class Hours/Week, 46 Students.

 

[Note:  No hour midterm exam questions found for the fall term 1965-66.]

Monday, January 24, 1966
Time: 1:30-4:30 p.m.

MASSACHUSETTS INSTITUTE OF TECHNOLOGY
Scheduled Examination in
INTERNATIONAL ECONOMICS – 14.581

NOTE: Students are not permitted to use any books, notebooks or papers in this examination. If brought into the room they must not be left on the desks

Answer Question 1 and 3 others–all of equal weight. 45 minutes each.

  1. Discuss the significance for the pure theory of international trade of two of the following assumptions:

1) two countries, two commodities, two factors
2) identical linear homogeneous production functions of the first degree
3) the labor theory of value
4) perfect competition in goods and factor markets
5) no transport costs.

  1. What are the effects of a tariff on the distribution of income between countries and within them?
  2. Comment at length on the Meade view that financial policies can be used to achieve internal balance, and exchange-rate variation to achieve external balance.
  3. Write an essay on the “gains from trade,” including, inter alia, a discussion on what countries gain, how much, and under what circumstances.
  4. Argue for or against discrimination in international trade, including, as one case, the customs union.

_____________________________

Fall Term 1966-67

14.581 International Economics. Professor C. P. Kindleberger with P. Bardhan, 3 Class Hours/Week, 39 Students.

Hour Test
14.581
December 1, 1966
10:30 a.m.

Answer one question under each of A and B (two in all, half hour each). Use a separate book for each question. Mark with your name and letter and number of the question.

  1. Describe in detail how a central bank can use forward exchange operations a) to protect its foreign exchange reserves in the event of capital outflow; and b) to gain reserves. What are the benefits of such forward operations? their limits?
  2. For 1964, 1965, and 1966 first nine months at an annual rate, the United States balance of payments showed the following data:
1964 1965 1966*
(in billions of dollars)
Gold sales -0.1 -1.7 -0.6
Liquidity balance -2.8 -1.3 -1.2
Official Reserve Transactions Balance -1.5 -1.3 +0.8

*First nine months of 1966 at an annual rate, seasonally adjusted except for gold sales.

Did the balance of payments improve or worsen each year? If one cannot say, what more would one need to be able to do so? Explain fully.

B

  1. Suppose you have a model with two countries, three goods, three factors, and internationally identical fixed-coefficients production functions for each good. What are the sufficient conditions for factor-price equalization in this model?
  2. In the usual two-by-two trade model if all of wage income is spent on one good and all of rental income from capital is spent on the other good, find out the conditions for uniqueness of static equilibrium in such a model.
  3. Take a small country in a large world with given terms of trade. Suppose in this country capital grows at a higher rate than labour and there is Hicks-neutral technical progress at a uniform rate in all the industries. What will happen to the wage rate and the rental rate on capital?

 

14.581T
24 January 1967
FINAL EXAMINATION

Answer question 1 or question 2 (one hour) and three others (forty minutes each)

  1. Compared to a pre-trade situation how will free trade affect income distribution in the trading countries in terms of the Heckscher-Ohlin model, comment on the assumptions of this model.
  2. What do you think are the most important limitations of the existing theory of international trade? Give suggestions, in as much detail as possible, about how you would go about removing one or two of them.
  3. Defend or refute the view of those who claim that free trade hinders rather than stimulates economic growth.
  4. What difference does it make to the impact of a tariff in general equilibrium what happens to the proceeds of the tariff?
  5. Comment at length on the usefulness of the purchasing-power parity theory.
  6. Suppose you have a country large enough to affect world prices. In that context comment on Samuelson’s proposition that “some trade is better than no trade.”
  7. In a standard two-sector two-factor neoclassical trade model with constant proportions of income being spent on each good, show how patterns of specialization will change with factor accumulation.
  8. Protectionists argue out — occasionally successfully — a case for government intervention, but a case for government intervention is not necessarily a case for tariffs. Illustrate with reference to the case of external economies in production.

Source:  M.I.T. Institute Archives. Charles Kindleberger Papers, 1934-1999. Box 22, Folder “Examinations 14.581, 1949-1966”.

Image Source: Charles P. Kindleberger from the MIT Museum.

Categories
Economic History Exam Questions Fields Harvard Statistics

Harvard. Division Exams for A.B., General and Economics, 1920

The Harvard Economics department was once one of three in its Division in the Faculty of Arts and Sciences. The Departments of History and Government shared a general division exam with the Department of Economics and also contributed their own specific exams for departmental fields. This post provides the questions for the common, i.e. general, divisional exam and all the specific exams at the end of the academic year 1919-20 for fields covered by the economics department.

_______________________

Previously Posted Division A.B. Exams from Harvard

Division Exams 1916

Division Exams, January 1917

Division Exams, April 1918

Division Exams May 1919

Division Exams 1931

Special Exam for Money and Government Finance, 1939

Special Exam Economic History Since 1750, 1939

Special Exam for Economic Theory, 1939

Special Exam for Labor and Social Reform, 1939

_________________________

DIVISION OF HISTORY, GOVERNMENT AND ECONOMICS

DIVISION EXAMINATIONS FOR THE DEGREEE OF A.B.
1919-20

DIVISION GENERAL EXAMINATION
[April 29, 1920.]

PART I

The treatment of one of the following questions will be regarded as equivalent to one-third the examination and should therefore occupy one hour. Write on one question only.

  1. Compare pamphleteering and propaganda as methods of exerting political influence.
  2. What effect has the establishment of standing armies and navies had upon (a) political and (b) economic organization of the state?
  3. Show how, and why, the following were adapted to certain stages of society: (a) feudalism; (b) gilds; (c) nationality; (d) industrialism.
  4. Trace the course and explain the significance of the development of maritime law.
  5. Contrast the Greek and Italian city states.
  6. What are the wastes of the present industrial system and how, if at all, are they to be eliminated?
  7. Comment on the following: “History embraces ideas as well as events, and derives its best virtues from regions beyond the sphere of state.”
  8. Discuss the problems involved in the economic rehabilitation of Central Europe.
  9. What are the rights of minorities and how are they best secured?
  10. Compare the foreign policies of France, Germany, and the United States during the nineteenth century.

PART II

The treatment of three of the following questions in Part II is required and will be regarded as equivalent to one-third of the examination, and should therefore occupy one hour. The three questions are to be taken from the Departments in which the student IS NOT CONCENTRATING; two questions from one of these Departments and one question from the other.

A. HISTORY

  1. Why did Voltaire characterize the Holy Roman Empire as “neither holy, nor Roman nor an Empire”?
  2. What do you regard as the six most important naval battles in the history of the world! When and where were they fought, and who were the victors and the vanquished in each?
  3. Give a brief account of the relations of the United States and Canada.
  4. What have been the principal issues involved in the struggle over Home Rule?

B. GOVERNMENT

  1. What was the political condition of European states at the time of the Crusades!
  2. In what sense are constitutions of states “made”?
  3. If the principle of reparation of governmental powers is correct, why has the English cabinet system been approved?
  4. Explain the reasons for immigration to the United States from 1870 to 1895.

C. ECONOMICS

  1. What has been the contribution of the corporation to English and American political and economic institutions?
  2. Trace the evolution of collective bargaining in industry.
  3. What is “profiteering”? Explain its relation to the present high cost of living.
  4. Describe the development, and indicate the importance, of national budgets.

PART III

The treatment of three of the following questions in Part III is required and will be regarded as equivalent to one-third of the examination, and should therefore occupy one hour. The three questions are to be taken from the Department in which the student IS CONCENTRATING.

A. HISTORY

  1. Describe the changes in the attitude towards the Christians of the Roman Emperors down to Constantine.
  2. Discuss the development of national assemblies during the Middle Ages.
  3. What did the Tudors do for England?
  4. What is now the territory within the jurisdiction of the United States has been derived, directly or indirectly, from seven European nations. What are the seven, and what territory was derived from each?
  5. Enumerate, with dates, the principal changes in the form of government of France since 1789. How do you account for their frequency?

B. GOVERNMENT

  1. Discuss the development of the relations of President and Cabinet in the United
  2. Discuss and illustrate the following: “If tolerance can be allowed in a state, so much the better; that proves that the state is strong.”
  3. What should be the disposition of Constantinople?
  4. Give a brief sketch, explaining cause and naming period, of three of the following: (a) Dorr Rebellion; (b) Whiskey Insurrection; (c) Shay’s Rebellion; (d) Seminole War; (e) Ku-Klux Klan.
  5. How has the change in distribution of population affected governmental organization and administration?

C. ECONOMICS

  1. Discuss the probable future of the market for loanable funds in the United States and Europe.
  2. State the purposes and proper limits of progressive taxation.
  3. Describe the efforts of the Federal Government to enforce fair competition.
  4. What considerations are involved in the maintenance of public agencies for the distribution and employment of labor? What light is thrown on the subject by American and European experience?
  5. Sketch the history and present prospects of the American merchant marine.

_________________________

DIVISION OF HISTORY, GOVERNMENT, AND ECONOMICS

DIVISION SPECIAL EXAMINATION
ECONOMIC THEORY
[May 3, 1920.]

Answer six questions

A

Take from this group at least two and not more than four

  1. “It is the business of economic theory to explain, not to justify or condemn.” Comment.
  2. Discuss the rôle of mathematical analysis in the development of economic theory.
  3. “The determining cause of the general rate of money incomes and wages in a country is to be found in the exporting industries.” Explain.
  4. “The income from concrete instruments of production may be regarded as ‘rent’ or as ‘interest’ according to the point of view.” Explain and discuss.
  5. Of what concretely do invested, of what do uninvested, savings consist? Can savings accumulate to an indefinitely large amount? Can saving be carried to excess?
  6. “The standard of living affects wages, not directly, but through its influence on numbers. … A limitation of numbers is not a cause of high wages, but it is a condition of the maintenance of high wages.” Explain and criticize.
  7. Discuss the theories of business profits.

B

Take from this group at least one and not more than two

  1. Outline the history of mercantilism.
  2. Give an account of an important political episode in which economic theory has had a decisive influence.
  3. Trace the course of the rate of interest in modern times. What do you expect to be the course of the rate during the next fifteen years? Why?
  4. Characterize the more important developments in the history of socialism.

C

Take from this group at least one and not more than two

  1. What is the relation of (a) investment banking, (b) commercial banking, to capitalistic production?
  2. What theoretical problems are involved in government regulation of security issues?
  3. Does profit-sharing promise a solution of the problems of distribution? Why or why not?
  4. Discuss the following statement: “If you are not advertising, then advertise, because it saves money for you and it reduces the price to the consumer.”

_________________________

DIVISION OF HISTORY, GOVERNMENT, AND ECONOMICS

DIVISION SPECIAL EXAMINATION
ECONOMIC HISTORY
[May 3, 1920]

Answer six questions

A

Take from this group at least one and not more than two

  1. To what extent, if at all, and in what particulars, has the policy of high protection been of advantage to the American laborer?
  2. How do price revolutions, such as that in progress since 1897, tend to affect the distribution of wealth?
  3. Briefly explain the most satisfactory statistical methods for separating the different types of variation in time series.
  4. What is a logarithmic curve? What are its merits and defects in the graphic presentation of historical series?
  5. Trace the development of uniform accounting for railroads in this country. Indicate any connections between uniform accounting and government regulation of the railroads.

B

Take from this group at least two and not more than four

  1. Discuss the economic results of the crusades.
  2. Give a brief historical account of mercantilism.
  3. Outline the history of the public debt of one of the following countries: (a) Great Britain; (b) France; (c) United States.
  4. Trace the agrarian movement on the continent of Europe.
  5. Discuss the positions of the various English political parties and social classes on the question of Corn Law Repeal.
  6. Write a brief history of one of the following industries in the United States:

(a) Meat-packing;
(b) Tin-plate manufacture;
(c) Boot and shoe manufacture;
(d) Ship-building.

  1. When and by what steps was silver demonetized in the United States?
  2. Outline the development of the English textile industry.
  3. Give a brief account of the “trust movement” in the United States since 1898.
  4. Sketch the history of the export trade of the United States.

C

Take from this group at least one and not more than two

  1. Analyze the effects of England’s early commercial policy.
  2. What specific defects in the National Banking System was the Federal Reserve Act, 1913 intended to remedy?
  3. Trace and explain the history of the American merchant marine since 1840. What is its probable future and why?
  4. What industrial conditions are most conducive to the rapid growth of labor organizations? Why?

_________________________

DIVISION OF HISTORY, GOVERNMENT, AND ECONOMICS

DIVISION SPECIAL EXAMINATION
MONEY AND BANKING
[May 3, 1920]

Answer six questions.

A

Take from this group at least one and not more than two

  1. Discuss the distinction between currency expansion and currency inflation.
  2. What statistics of money and banking best serve as indices of financial, speculative, and general business conditions?
  3. Outline a system of accounts for a small commercial bank.
  4. What are the best sources of statistical data upon the following subjects:

(a) Bank clearings in the United States;
(b) Resources and liabilities of banks of New York City;
(c) Bank rates in the London and Paris money markets;
(d) The monetary stock of the United States;
(e) Changes in the value of gold in England?

B

Take from this group at least one and not more than two

  1. Outline the currency history of one of the following:

(a) Canada;
(b) Germany;
(c) British India;
(d) the Philippines;
(e) the American colonies;
(f) Russia.

  1. State and explain Gresham’s Law. Give four historical examples of the working of the law.
  2. Sketch the history of the relations between the United States Treasury and the banking institutions of the country.
  3. Compare American, British, and German banking methods and policy during the World War.
  4. Describe in detail one of the following financial panics: 1837; 1873; 1893; 1907.

C

Take from this group at least two and not more than four

  1. What have been the causes of the rehabilitation of silver?
  2. What are the arguments for and against an embargo upon gold exports from the United States at this time?
  3. Describe the business of an American bond house.
  4. Discuss critically the following statement made early in 1916:
    “The recent enactment of the Federal Reserve Act only made our sudden riches more embarrassing, for that Act had so changed our system of banking that every $18 of gold in the banks created $82 worth of loanable credit, whereas formerly, of every $100, $25 had to sit in the vaults while only $75 went out to work in the form of loans. In other words (as a result of the War and our banking reform), we not only had enormously more gold, but every dollar of it went a good deal further than ever before in financing new enterprises. This is the situation today.”
  5. Give a critical analysis of the policies of the Federal Reserve Board.
  6. Compare banking in France and England since the signing of the armistice.
  7. Why has London been the financial center of the world? What are the prospects that New York will in time displace London?

_________________________

DIVISION OF HISTORY, GOVERNMENT, AND ECONOMICS

DIVISION SPECIAL EXAMINATION
CORPORATE ORGANIZATION, INCLUDING TRANSPORTATION
[May 3, 1920]

Answer six questions

A

Take from this group at least one and not more than two

  1. In discussing the problems of capitalistic monopoly, it has been stated that “the matter at issue is a question, less of relative ‘economy’ of monopoly and competition than of the kind of economic organization best calculated to give us the kind of society we want.” Explain and discuss.
  2. What are the methods of measuring depreciation? What different policies with respect to depreciation have been advocated in the regulation of public utility rates?
  3. Discuss comparatively the public regulation of railway accounts in England, France, and the United States.
  4. To what extent do the reports of the Bureau of the Census furnish data upon corporate enterprise in the United States?

B

Take from this group at least one and not more than two

  1. Sketch the history of the Sherman Anti-Trust Law and its enforcement.
  2. Give a brief account of the functions and work of the United States Bureau of Corporations.
  3. Trace the evolution of the equipment of the modern railway.
  4. Outline the history of railroads in Germany.

C

Take from this group at least two and not more than four

  1. What are the purposes and customary scope of “blue sky” laws? What is the case for and against such legislation?
  2. What connections exist between banks and industrial combinations in the United States? Contrast the situation here with that in France.
  3. Compare American and German public policy toward industrial combinations.
  4. Give a critical analysis of the present railway rate structure in the United States.
  5. Discuss the Plumb Plan for the ownership and operation of the railways of the United States.
  6. Discuss the effects of the great inter-oceanic canals upon inland and ocean transportation.
  7. What are the problems of excess profits taxation?

_________________________

DIVISION OF HISTORY, GOVERNMENT, AND ECONOMICS

DIVISION SPECIAL EXAMINATION
PUBLIC FINANCE
[May 3, 1920]

Answer six questions.

A

Take from this group at least one and not more than two

  1. To what extent, if at all, and in what manner, are taxes a contributing cause of the present “high cost of living”?
  2. Discuss the proposal to tax individuals in proportion to their expenditure rather than their income, thus exempting savings.
  3. Describe the statistical features of the Census Bureau’s annual reports on “Financial Statistics of Cities.”
  4. What course has been taken by the reform of municipal accounting in the United States?

B

Take from this group at least one and not more than two

  1. Outline the development of the science of public finance.
  2. Give a critical account of the Independent Treasury of the United States.
  3. Trace the history of budget plans in American state and municipal government.
  4. Compare the financing of the American and French Revolutions.
  5. Give a brief historical account of direct taxation in Germany.
  6. Develop and defend a classification of public revenues.

C

Take from this group at least two and not more than four

  1. Compare government monopolies and internal revenue taxes as means of raising national funds.
  2. Analyze the financial results of the operations of the United States Post Office.
  3. Upon what bases should public utilities be valued and paid for when taken over by municipal authorities?
  4. “Taxation, while necessarily involving political and social considerations, is essentially a problem in national economies.” Do you agree? State your reasons.
  5. The practice of exempting government bonds from taxation is a pernicious American custom.” Comment.
  6. Discuss the effects of national prohibition upon public finance.
  7. Give a critical analysis of excess profits taxation.

_________________________

DIVISION OF HISTORY, GOVERNMENT, AND ECONOMICS

DIVISION SPECIAL EXAMINATION
LABOR PROBLEMS
[May 3, 1920]

Answer six questions

A

Take from this group at least one and not more than two

  1. Discuss the causes of the prevailing industrial unrest.
  2. To what extent and for what purposes should the state limit the hours of labor?
  3. Describe the technique of analyzing workingmen’s budgets.
  4. What statistical problems are involved in measuring labor turnover? What methods of measurement are most satisfactory?

B

Take from this group at least one and not more than two

  1. Compare trade unions and trade gilds, and the industrial conditions under each.
  2. Give a brief historical account of the employment of children in industry.
  3. Outline the development of the Railway Brotherhoods.
  4. Trace the history of the German Social Democratic Party.

C

Take from this group at least two and not more than four

  1. Analyze labor conditions in one of the following industries: (a) cotton manufacture; (b) coal mining; (c) steel manufacture.
  2. Discuss the main points of economic policy in the “reconstruction program” of the British Labor Party.
  3. What is the extent and importance of industrial unemployment?
  4. Discuss the present status of women in industry.
  5. Characterize the organization and results of the Washington industrial conferences of 1919-20.
  6. Discuss the aims, scope, and methods of employee representation in business management.
  7. What public policy should be adopted in regard to labor organizations among government employees?

_________________________

DIVISION OF HISTORY, GOVERNMENT, AND ECONOMICS

DIVISION SPECIAL EXAMINATION
ECONOMICS OF AGRICULTURE
[May 3, 1920]

Answer six questions

A

Take from this group at least one and not more than two

  1. To what extent are wages of management an element of cost in American agriculture?
  2. What are the interrelations of cold storage and prices of farm products?
  3. What statistical records are desirable for efficient operation of a dairy farm?
  4. To what extent and in what particulars is depreciation involved in farm accounting?

B

Take from this group at least one and not more than two

  1. Describe the agrarian revolution in England.
  2. Sketch the movement of the wheat belt in the United States since colonial times.
  3. Give a brief historical account of farm tenancy in the United States.
  4. Trace connections between the tariff policy of the United States and wool growing in this country.
  5. Outline the development of the work of the United States Department of Agriculture.

C

Take from this group at least two and not more than four

  1. Discuss the relations between climate and the productivity of land.
  2. Indicate the origins of the more important breeds of live stock. What contributions, if any, has this country made to the improvement of the breeds?
  3. Describe the effects of the World War upon the wool market.
  4. What are the relations between the wages of agricultural and factory labor?
  5. Compare the use and importance of artificial fertilizers in American and European agriculture.
  6. Give a brief critical analysis of the Federal Farm Loan Act.
  7. What are the causes of the increasing urbanization of population in the United States?

_________________________

DIVISION OF HISTORY, GOVERNMENT, AND ECONOMICS

DIVISION SPECIAL EXAMINATION
STATISTICS
[May 3, 1920]

Answer six questions

A

Take from this group at least two and not more than four

  1. Draft a set of rules for the construction of statistical tables.
  2. Explain “the necessity of the logical agreement of magnitudes from which an average is to be computed” and compare this with the requirement of “the greatest possible homogeneity of series.” Which of these requirements seems to you more difficult of fulfilment? Why?
  3. Describe the short-cut method of calculating the arithmetic mean from a frequency table. What assumptions underlie this method?
  4. Explain briefly: (a) discrete series; (b) mode; (c) Lorenz curve; (d) average of position; (e) Galton graph.
  5. Explain the different methods of eliminating secular trend in historical series.
  6. Describe the construction and characteristics of a logarithmic curve. What are the merits and defects of such a curve?
  7. What are the comparative advantages and disadvantages of chain indices and fixed-base indices in the measurement of changes in the price level?

B

Take from this group at least one and not more than two

  1. Give a brief account of the evolution of Statistics.
  2. Outline the history of the Bureau of the Census.
  3. Trace the development of price statistics in England.
  4. What has been the history of wage statistics in the United States?

C

Take from this group at least one and not more than two

  1. Discuss the different methods employed in estimating population.
  2. What are the principal difficulties in the collection of mortality statistics?
  3. Discuss critically current statistics of foreign trade in this country and abroad.
  4. What units have been employed in the statistics of railways? Analyze and appraise the different units.
  5. What are the best sources of statistical data upon the following subjects:

(a) Bank clearings in the United States;
(b) Resources and liabilities of banks of New York City;
(c) Bank rates in the London and Paris money markets;
(d) The monetary stock of the United States;
(e) Changes in the value of gold in England?

 

Source: Harvard University Archives. Harvard University Divisional and General Examinations, 1915-1975. Box 6, Bound volume Divisional Examinations 1916-1927 (From the Private Library of Arthur H. Cole).

Image Source: Widener Library from Harvard Class Album 1920.

Categories
Chicago Exam Questions

Chicago. Exams for second graduate price theory course. Griliches, 1965

 

A few posts ago Economics in the Rear-view Mirror presented the exams for the first quarter of graduate price theory (Economics 300) at the University of Chicago taught by Giora Hanoch in the autumn quarter of the 1964-65 academic year. In this post we have the exam questions for the winter quarter’s second graduate price theory course taught by Zvi Griliches.

As I transcribe these mind-numbing true-false-uncertain questions, I have wondered if there ever was a University of Chicago graduate student who answered all of the questions “uncertain” and tortured the graders with special cases, counter-examples, and intricate ad-hoc-ceteris-not-so-paribus explanations. But then I think of the canonical image of a WWII bomber that has returned to base with flak damage. Goodnight Mrs. Calabash, wherever you are.

________________________________

ECONOMICS 301
February 10, 1965
Two-hour Midterm Examination

I. (70 points)

Answer whether the statement is true, false, or uncertain. In each case, write a few sentences explaining your answer. Your grade will depend heavily on your explanation.

  1. The elasticity of a linear supply function that passes through the origin is always unity.
  2. If a firm is producing in the region of rising marginal costs, the firm is realizing profits.
  3. An effective price ceiling on cotton, i.e., one that holds its price below the free market level, will decrease the price of textiles.
  4. Steel prices and output usually move together during business cycles. This means that the income effect of a rise in price is greater than the substitution effect.
  5. Firms try to minimize unit costs; at the point where unit costs are at a minimum, they equal marginal costs; therefore, firms tend to operate where their unit and marginal costs are equal.
  6. Marginal productivity theory does not apply if factors are always used in fixed proportion.
  7. Since all firms in competitive industry have the same marginal costs, it is meaningless to speak of more or less efficient firms.
  8. If a Paasche price index is higher than the Laspeyres’ index, tastes must have changed.
  9. The demand for a product at the market price is inelastic. It follows that the product must be produced under conditions of net internal diseconomies.
  10. “Commodities with higher, income elasticities have higher demand (price) elasticities.” (Stigler, 1952 ed., p. 45)
  11. If X and Y are substitutes, a decline in the price of X can increase the amount of Y demanded only if Y is an inferior good.
  12. The elasticity of demand for a group of commodities with respect to the average price of the group can never be larger in absolute value than the largest of the individual price elasticities of the commodities which comprise the group.
  13. A rational consumer is insatiable.

II. (30 points)

A. The demand function for a product is P = 115 — Q. The total cost of producing Q units in one plant is given by TC = 400 — 100Q2 + Q3. Only one-plant firms are allowed.

(a) What is the long run competitive solution (price, quantity, and the number of firms in this industry)?

(b) What would be the approximate price charged and the quantity produced if there was only one one-plant firm and it maximized its profits. (Work only with round figures.) How much profit would it make?

B. Assume now that a firm may have more than one plant. What is the monopoly solution? How much profit will it make?

________________________________

March 15, 1965

ECONOMICS 301
Z. Griliches

FINAL EXAMINATION
Winter, 1965

2 HOURS TO COMPLETE EXAM

I. (80 points)

Answer each question “true”, “false”, or “uncertain”, and explain your answer briefly. Your grade will depend heavily on your explanation.

  1. A competitive firm will increase output as the result of a fall in the price of one of its inputs.
  2. In equilibrium, a competitive firm has all the business (sales) it wants. Hence advertising is incompatible with either competition or equilibrium.
  3. Duopolists with different cost functions cannot achieve a monopoly price without transfer payment between the firms.
  4. A multiplant firm will schedule its output so that the marginal costs are equal in all plants.
  5. The price of haircuts in Chicago is approximately 40 percent higher than in New York; therefore, average earnings of barbers in Chicago are higher than in New York.
  6. The supply curve of a monopolist is inelastic at the point of maximum monopoly profit.
  7. If it takes one day to catch a beaver and two to catch a deer, one deer will exchange for two beavers.
  8. Assume that the world demand elasticity for tin is -2 and that Bolivia produces 1/3 of the world’s tin. Therefore, the elasticity of demand for Bolivian tin is at least -6.0 (in absolute value).
  9. A safety ordinance prohibiting the use of automobiles older than 10 years will increase the long run demand for new automobiles.
  10. The own-price elasticity of demand for a commodity is no smaller in absolute value, than the marginal propensity to consume that commodity.
  11. For a single consumer the sum of income elasticities of demand for all commodities is unity, while the sum of their price elasticities is zero.
  12. It is a convention in economics to draw consumption indifference curves convex to the origin, but we have no way of knowing whether they really are.

II. (10 points)

Each firm in an industry is given a license to operate and no new firms are allowed to enter. The value of a license rises over time. Does this prove that firms operate subject to diseconomies of scale?

III. (30 points)

It is often asserted that Americans love money more than Englishmen (or Europeans, or Latin Americans). Can you think of a way to test this proposition?

Source: Harvard University Archives. Papers of Zvi Griliches. Box 130, Folder “Syllabi and exams, 1961-1969”.

Source: From an image of the Brazilian immigration/visa card for Zvi Griliches dated 18 Aug 1959 that can be found at the ancestry.com website.

Categories
Columbia Cornell Economics Programs Harvard Michigan Popular Economics Yale

“Political Economy and the Civil War” by Laughlin that provoked an Economist-Bashing editorial, 1885

Before becoming the founding father of the department of political economy at the University of Chicago, the 35 year old Harvard assistant professor J. Laurence Laughlin (Harvard Ph.D. 1876) published an essay, transcribed below, arguing that liberal college education needed to be expanded beyond Greek, Latin, mathematics, and philosophy to include courses dealing with economic theory and its policy applications. He provides us a table of the limited course offerings in political economy at five major colleges/universities at the time. I stumbled upon an unsigned editorial written in response to Laughlin that I have also transcribed and which is placed at the end of this post. The editorial provides us with historical evidence that ill-tempered economics-bashing is hardly a creation of the Twitternet Age. No siree Bob! The editor was not amused by Laughlin’s presumption, calling him and his college professor colleagues who taught political economy to boys…”vealy milksops”. I dare any or all visitors to sneak that expression into a footnote.

________________________________

POLITICAL ECONOMY AND THE CIVIL WAR.
By J. Laurence Laughlin.

Atlantic Monthly, v. 55 (April, 1885) no. 330, pp. 444-450.

In some parts of our country there is a current maxim among the old-fashioned gardeners to the effect that “a wind-shaken tree will bear much fruit.” There is some subtle force in it. In fact, it is an expression which may be regarded as finding its parallel in individual and social life. As individuals, we know that there is no real growth of character except by a conquest over opposing difficulties; the doing right when it is against our inclinations and prejudices. And in a social organism we seem to see a moral law of conservation of energy by which a sacrifice is the parent of some gain,— a thing which evidently underlies the movements attending many great convulsions in political life. We saw armies go out of our sight during the civil war, only to come back thinned, injured by disease, with half their number left dead on the field. Death meant bitter, indescribable sorrow in all our homes. The experiences of the war were felt to be pitiless, inexplicable, and hard. And yet, perhaps, a subtle suggestion may have come into our minds that it was not simply by dying, or in living, that the best law of our being was enforced; that there was, in truth, some Power behind it all; that some purpose was being worked out through each one of us, just as each leaf on the tree, for example, is necessary to the completed organism of the whole tree, and ceases to be when it is separated from the stem. Now, perhaps, even at this short distance from the struggle, we can begin to see some of the effects of that social and political upheaval, the greatest since the foundation of our government. It is worth while to examine whether the wind shaken tree has borne much fruit.

The process by which citizens from the secluded districts and remote towns were sent through new cities to opposite parts of the Union, exchanging ideas with men of different habits of thought, was a marked feature of the war period, and leavened the mental life of the American people in a way hither to little suspected. It was something like sending a country boy to college, only the effect was multiplied a million times. The rural population came into a knowledge of our cities, while the urban classes were carried out into new climates and into unvisited parts of our vast domain. New sights, new methods of cultivation, different habits of living, stimulated the dull and fired the active and enterprising men in the ranks. The life of the farm and the village was widened to an interest in the nation. About the same time, moreover, came a vast increase in easy means of communication by railways and a greater extension of the use of the newspaper and telegraph, by which provincial towns were brought into direct connection with the outside world. Even oddities of customs and dress began to disappear, in the process of comparison with the more attractive ways of the dwellers in the great cities and towns. In this fashion, the thinking horizon was extended. Dull intellects learned the presence of complicated problems, and brighter minds found new spurs to ambition in the questions of larger interest. On all sides men felt themselves coming daily into contact with new difficulties, under a dim consciousness of their bigness, but with a strong belief that the knowledge how to deal with them was inadequate. In short, the tremendous crisis through which we passed, apart from its effect on the preservation of the Union, has been subtly at work in moral and intellectual directions. The working of these new forces on a quick and susceptible race can easily be imagined. They have, in fact, under somewhat similar conditions, had a distinct influence on a more phlegmatic people than ours. Old students at Göttingen, who have returned to the university since the late wars in which Germany has been engaged, have been amazed to find the old-fashioned spot — where the customs, habits, and naive simplicity of one hundred years ago had prevailed until quite recently — now wholly changed. The commercial spirit has seized the formerly simple-minded peasants, and the quiet town now hears the heavy march of cosmopolitanism in its streets.

Like Germany, the United States had new problems to solve. While the conflict closed the long slavery struggle, it brought with it intricate questions, but of a character very different from those which had gone before. Without warning, and consequently without the ability to get due preparation or acquire proper training, our public men were confronted, as the war progressed, with matters of vital importance in international and constitutional law, in taxation, and in every form of administration and finance. The demand for men who had given themselves more particularly to the province of governmental science was an imperative one; but it was, generally speaking, met in a way which showed that there existed in the community a class from whom these necessary men could be recruited. That class was the legal profession of the country. The questions of reconstruction, the relation of the general government to the States, the civil rights of the negro, our relations with foreign powers during the blockade of Southern ports, were not abandoned to men who had never habituated themselves to discussions such as were involved in their settlement. There were differences of opinion, of course; but inasmuch as these differences of opinion were produced by different political theories, this proved that attention had been given to such subjects to the extent that a crystallized system of thought, formulated in dogmas, had been created by the various parties.

But, as has been suggested, new considerations arose. The magnitude of the military operations involved an enormous expenditure of money by the state, and made a demand upon our statesmen for financial skill of an almost unparalleled kind. To meet these extended questions of taxation, finance, and currency, what body of men could be called upon? To this, answer must be made that the war overtook us without a supply — or even a few — of trained economists and financiers. The economic part in the equipment of a public official had been wholly neglected. In fact, political economy and finance had never been seriously studied in the schools; but, if studied, they were classed in the old-fashioned required curriculum with Butler’s Analogy and the Evidences of Christianity. Although Adam Smith wrote his Wealth of Nations in 1776, political economy was an unknown science to the American people before 1860. It is an interesting study to examine the manner in which our people went under the burdens and tasks of our great civil conflict. There was the quick adaptability of Americans to start with; there was plenty of patriotism and good will, and no lack of those high qualities of self-sacrifice and heroism which are still fragrant to us; but lawyers, such as Chase and Fessenden, were practically our only financiers. Early in the war they were required to consider a scheme — for the right settlement of which a vast experience is necessary — of raising loans, and adjusting a plan of taxation corresponding to the extraordinary war expenses. Without considering alternatives, in a few years they created a debt as great as that incurred by old despotisms of Europe in centuries; without foresight, they drifted into a ruinous issue of irredeemable paper money; without intending it as the object of a definite policy, but through a desire simply to gain a war revenue, they established an extended system of “protection to home industries” by levying duties on imports, which has brought into existence business interests largely dependent on the continuance of these temporary war measures. When it is realized that principles of taxation are to-day probably less understood than any other branch of economics, it is not surprising to find that in 1864 Congress was occupied only five days in passing the most gigantic taxation measure of the war. The National Bank Act, which has given us the best system of banking ever enjoyed by the country, was, however, in reality passed as an act to facilitate the sale of our bonds and aid our tottering credit. We blundered egregiously, but we were capable of learning by experience. Yet it was from these very blunders, from this revelation of inexperience made evident by the demands of a great emergency period, that the community received an impetus toward the study of economic questions which was certain to result in good fruit.

In fact, it is now clear that a new interest in economics and finance has already arisen. The civil war was, so to speak, the creation of economic study in the United States. The war did for this country — in a different way, of course — even more than the corn-law agitation did for England. It actually gave birth to new motives for study. There never was a time in our history when there was so evident a desire to get light on the economic problems of the day as now. There is a new stir among the ranks of the young men at college; and the printing-press sends forth an increasing stream of new books upon subjects which are constantly discussed in the daily newspapers. There is unquestionably a new-born, slowly growing attention by the younger men of our land to the necessity (as well as the duty) of fitting themselves properly for the responsibilities of citizenship. If the war has given us this, — the absence of which used to be so often lamented a few years ago, — then may some of our sacrifices not have been in vain. The wind-shaking has resulted in abundant fruit.

In the present awakening in educational discussion, one phase of which has been called the “Greek Question,” it is worth while to notice the influence of the war period on the college curriculum. In most of our schools and universities, on the breaking out of the war (and even to the present day), the pecuniary resources and endowments had been tied down, under the force of old traditions, to supply instruction in the customary Greek, Latin, mathematics, and philosophy, which were then considered the only essentials of a liberal education. But when the rude shock of the war awakened us to our ignorance, and we looked around for the schools where the new studies could best be followed, it was discovered that the college curriculum made practically no provision for such instruction. In the old days when sailing vessels alone entered Boston harbor, only one channel was practicable, and all the fortifications were placed in a way to command it: but when steam took the place of sails, another channel was adopted, but it is now wholly undefended. The old ship channel must be defended, but so must the new one. So, in the collegiate studies, the old subjects are necessary, of course, but they are not the only necessary ones. The new demands, due to the progress of the age, must also be met. In fact, the response of the schools to these new demands is at once the evidence and result of the quickening and stimulating forces so briefly sketched in these pages. A comparison of the amount of instruction in political economy given by the principal institutions of the land in the years 1860, 1870, and 1884 will furnish us new proof that the wind-shaken tree is yielding full fruit.

Nothing could show more distinctly than the accompanying table how young any real systematic study of political economy is in this country, and it accounts for the lack of any number of trained economists among us. But the younger generation are happily recruiting their ranks, now that these better opportunities are open to them.

At no time, however, have public affairs demanded unpartisan study in economics more than to-day. In past centuries governments were supposed to labor, in an unsettled state of society, for the protection of life and property. Now that the general progress of civilization and Christianity has made life and liberty more secure, legislation in later years has concerned itself rather with property than life. In the Middle Ages trade was considered plebeian; to fight or to oppress was regarded as more noble. Now the chief solicitude of the modern state is the increase of wealth: the castles have become mills; retainers, productive laborers; and arms, the hammers and tools of the artisan.

1860.

1870.

1884.

Yale College. One third of Senior year One third of Senior Year 1.  Elementary Course. — Fawcett. — Discussions on currency, banking, and taxation. 3 hours a week for 13 weeks.
2.  Elementary Course. — Mill. — Currency, banking, and taxation. 2 hours a week for a year.
3.  Advanced Course. — Discussion of economic problems and fallacies, with selections from leading treatises. 2 hours a week for 20 weeks.
4.  Graduate Course. — Finance and the Art of Politics, as illustrated in the History of the United States. 2 hours a week for 2 years.
5.  Graduate Course (in alternate years.) — In 1883-4, Sociology. In 1884-5, Industrial History, History of Political Economy, Finance and Theory of Rights. 1 hour a week for each year.
6.  History, business methods, and social problems, of Railroads. 2 hours a week for a year.
[A course about equal to Courses 1 and 2 is given in the Sheffield Scientific School.]

Cornell University.

[Institution not founded]

One third of Junior Year

1. Elementary Course. — Lectures and Recitations. 2 hours a week 2/3 of a year.
2. Lectures on Political Economy.5 hours a week for 1/3 of a year.
3. Lectures on Finance.

University of Michigan.

Not in the Course of Study.

One Term of Senior Year.

1. Elementary Course. — Lectures. 3 hours a week ½ of a year.
2.  Advanced Course. — Competition, Free Trade and Protection, Commercial Depressions, Transportation, etc. 3 hours a week ½ of a year.
3.  Principles and Methods of Finance. — Banking, National Debts, etc. 2 hours a week ½ of a year.
4.  History of Industrial Society [not given in 1883-4]. 2 hours a week ½ of a year.
5.  Financial Seminary.— History of American Finance. 2 hours a week ½ of a year. [Not given 1883-4.]

Columbia College.

Elective in one part of Senior Year.

One Term of Senior Year.

1.  Principles of Political Economy.— Elementary Course. Rogers’ Manual. 2 hours a week ½ of a year.
2. History of Politico-Economic Institutions. 2 hours a week ½ of a year.
3.  Finance and Taxation. 2 hours a week ½ of a year.
4.  Statistical Science, Methods and Results. 2 hours a week ½ a year.
5.  Communistic and Socialistic Theories. 2 hours a week ½ a year.
6.   [Topics like railways, banks etc., are placed under Administrative Law.]

Harvard University.

One half of Senior Year.

1. Rogers’ Manual One half of Junior Year 1. Elementary Course.— Mill’s Political Economy. Lectures on Banking and the Financial Legislation of the United States. 3 hours a week for a year.
2. Elective Course for Seniors: Adam Smith, Mill, Bowen. 3 hours a week for a year. 2. Advanced Course.— History of Political Economy. Cairnes, Carey, George, and recent literature. 3 hours a week for a year.
3. Investigation of Practical Questions of the Day.— Banking, Money, Bimetallism, American Shipping, Note Issues, etc. 3 hours a week for a year.
4. Economic History since the Seven Years’ War.— 3 hours a week for a year.
5. Land Tenures in England, Ireland, France, and Germany.— 1 hour a week for a year.
6. History of Tariff Legislation in the United States.— 1 hour a week for a year.
7. Comparison of the Financial Systems of France, England, Germany, and the United States.— 1 hour a week for a year. [Omitted 1884-5.]
8. History of Financial Legislation in the United States. 1 hour a week for a year.

Consider the character of the questions at this time pressing upon Congress for immediate attention. If we omit the administrative and political legislation on the civil service, the succession to the presidency, and a national bankruptcy law, the remaining questions before Congress to-day are almost entirely economic. (1.) There is, in the first place, the false silver dollar, masquerading in sheep’s clothing, and waiting to catch the unwary business world napping, when it will suddenly assume its true depreciated character, and devour fifteen or eighteen per cent of all creditor’s dues estimated at present prices. What is Congress doing here? Just what it did in the last months of 1861, when it let the country drift on to the shoals of depreciated paper. Monometallists and bimetallists, business men and bankers, are assaulting the dangerous silver legislation, and yet Congress is a very Gibraltar in which the silver owners are intrenched. (2.) Next, there is the banking question. Nothing can be more delicate and sensitive than the machinery of credit and banking in a great commercial country such as this; and yet men, to satisfy the prejudices of constituents, handle this mechanism with about the same air of cheerful indifference as that of a child who drags a rag doll round by the heels. The present national bank notes give a stability to trade in separate parts of the Union, by means of a currency equally good in Maine and Texas, never reached in the days of the vicious and changing state banks; and yet the present system is gradually vanishing before our very eyes, as calls are made for government bonds. (3.) Again, Congress is struggling with the most difficult of all problems, – national taxation. It means a reëxamination of our whole scheme of taxation, the retention of internal taxes on distilled spirits and tobacco, the management of our surplus revenue, the whole sub-treasury system; while the situation inevitably requires a readjustment of our customs duties. Duties needed in order to procure a large revenue in time of war are no longer necessary when the war is ended, and the national debt is reduced one half. (4.) There are the barbarous and mediaeval navigation laws, to which we cling with a curious indifference to the influence of all progress and liberal ideas. The problem of our shipping and merchant marine needs the touchstone of some wider training than is furnished by selfish individual interests. (5.) Our public lands and the settlement of our vast Western domain are important matters of land tenures, and yet they are abandoned to accident, while the possibilities of good disappear under the cloud of accomplished facts, where nothing can be done. It will not be long before all the public lands will be gone, and yet no notice is taken of existing evils. (6.) Then, again, one has but to mention the word “railway,” and there arises to the mind a congeries of difficult questions dealing with Western “grangers,” the ability of the state to regulate freight and passenger charges, and in fact the whole vexed discussion of state interference. Here is a field by itself, to which a man may well give his whole life-work. (7.) It would be wearisome to more than mention the topics of Postal Telegraph, Chinese Labor, Strikes, Trades Unions, and Communism, which attract our instant attention. (8.) Then again the unfortunate legal-tender decision of Judge Gray has brought back to us all the troublesome and intricate discussions on the currency which we once thought had been forever settled. As matters now stand, power is given to Congress, if it chooses, to repeat all the errors of Continental currency policy, and we are put back a century in our paper money teaching. (9.) To pass from merely internal matters, so long as we were the only civilized people on the western continent, our relations with our neighbors gave us little thought. The growth of commerce, the expansion of populous areas north and south of us, the discovery of mineral wealth outside our own limits, which invites our capital, has forced on us the consideration of reciprocity with Canada and Mexico. We have refused reciprocity to Canada; but to-day we are considering the desirability of granting closer commercial relations with Mexico, while Cuba and Porto Rico have asked the same advantages by a new treaty.

Such, in brief, are some of the subjects which must be made matter of instruction in our schools and universities. It will be observed how overwhelming a proportion of public measures at present are economic, and what a heavy responsibility lies upon our institutions of learning, if they are to meet the new demands in a fitting manner. But there is a still stronger reason for strengthening our educational forces on the economic side. This is to be found in what may be called the “economic portents.” To the present time we have been properly called a “young country,” which to the economist means an abundance of unoccupied land, a scanty population, large returns to capital, and high wages. A full knowledge of our resources has not practically been reached as yet, and will not be, probably, for a considerable time to come. These resources and the lusty health of our young country have made it possible heretofore for legislators to blunder with impunity. Constantly receiving large returns, labor and capital would not naturally be over-critical and hostile to each other. The young-country theory has also led to the encouragement of unlimited immigration, with which to settle our prairies and build up our towns. These new-comers do not, in fact, all go upon the land; but, arriving on our seaboard, instead of being drawn off entirely, they remain in the cities, like dirty pools of water in the streets. Indeed, the importation of uneducated, un-American, un-republican workmen from foreign lands is a problem in itself, and makes a strong demand upon all who can possibly do so to educate these masses, both economically and politically. Lawless communism, it is said advisedly, feeds on bad workmen. A saving mechanic is never a communist. To-day these men mean little to us; but when, by an increasing population and a denser settlement of the country, land becomes more scarce and valuable, profits on capital lower, and wages less, then even honest men, finding themselves pinched by a barrier of their own creation, brought into operation by natural laws, unless economically trained, will not know what is happening, and may in entire ignorance fly in the face of the law, and do in the United States somewhat of the things they are now doing in Europe. The day is more or less distant when this may happen, but it is coming nearer in proportion as the methods of men accustomed to conditions in old and crowded countries are brought here by a never ending stream of immigration.

The war has plunged us into the consideration of gigantic questions of an economic character, and the growth of our country in numbers and wealth is making a true understanding of them more necessary than ever to the prosperity of the nation, and a rising tide of interest in such studies is unmistakably evident. But these new and increasing demands are met by meagre and inadequate means in the great schools. It is a surprising fact that in some of the most important institutions there is no separate provision for such studies, and not even one settled instructor. Above all, we must educate in an intelligent manner, by stimulating investigation into home problems, and by encouraging the preparations of monographs on some out of the multitude of our economic questions. The best of the men in the university cannot now find a career in economic teaching, because few positions exist in this country as an object for honorable and ambitious students. Men find a profession in teaching Greek and Latin, but not Political Economy. When the community wakes up to a realization of this gap in the instruction of the land, and the importance of filling it, we may hope to see a more correct relation between means and needs than now exists.

________________________________

COLLEGE PROFESSORS AS ECONOMISTS.

Mr. J. Laurence Laughlin, in the Atlantic Monthly for April, appends his name to one of those egotistical screeds which serve to make those who teach political economy to boys contemptible in the sight of those who have occasion to practice legislative economies as practical statesman. Its fundamental assumption is that for want of the wisdom with such boys as Laughlin and Sumner possess nearly all that Hamilton, Gallatin, Chase, and Fessenden have done in America and quite all that Colbert, Napoleon, Pitt, Turgot, and Bismarck have done in Europe in an economic and financial way has been sad botchwork. Why do magazines like the Atlantic Monthly publish such ridiculous rant?

Instead of Chase and Fessenden having been in need of going to school to such vealy milksops as J. Laurence Laughlin, this college tutor shows on every page that he writes how greatly he needs the practical information which he could have got by attending for two or three years on the sessions of the Ways and Means Committee at Washington. Indeed, it is not legislators that need to be educated in economics by college professors, but college professors who need some means of picking up a few grains of sense by being brought into contact with actual legislation.

It is a singular fact that no man who has ever accepted a chair in a college as a teacher of political economy to boys has ever yet rendered any demonstrable service either to the cause of economic science or of legislation. Laughlin has the impertinence to say that, though Adam Smith wrote his “Wealth of Nations” in 1776, political economy was an unknown science to the American people before 1860. Does Mr. Laughlin mean to assert that Franklin, the intimate personal friend of Adam Smith and suggestor of some of his views, or that Hamilton, Madison, Jefferson, Clay, or Webster, Chase, Fessenden, Garfield, or “Pig Iron” Kelly are any less familiar with Adam Smith’s crudities, blunders, wisdom, and garrulous mud than Laughlin himself is? Adam Smith fell so far below Alexander Hamilton, and in many respects below Madison and Chase, in economic insight that while every commentator on Smith points out errors of fact and of theory, stupidities of ignorance and obliquities of vision on every page of the old scotch dullard and mugwump, we challenge Laughlin to point out with equal ease the ignorances and blunders in Hamilton’s economic papers or financial reports.

Adam Smith had the merit, however, of only styling his work as an “Inquiry.” It is the men who come after him who arrogate for his utterly unscientific, undefined meandering, inconsequential and self-contradictory fog-banks the quality of a science. Still Smith is helpful matter to a sensible legislator, because the latter can generally see on the face of Smith’s statements wherein the good Scotch plodder was wool-gathering, and could rectify Smith’s errors out of his own more modern and ample reading. The notion however, that Cairnes, Mill, Jevons, McLeod, Say, Lavelaye, or any other boy teachers have ever been helpful in matters of practical legislation is not warranted by facts. Ricardo was listened to with great respect by practical legislators, but he was a practical businessman like Franklin, the Careys, and Greeley, who had never undertaken the egotism of a pedagogue. The only economists America has yet produced are those who have either never or hardly ever sat in a professor’s chair. There seems to be something in the air of a school room which, if the professor remains in it until it conquers him, unfits him absolutely to mingle as a man among men in the affairs of men. It causes a cranky adoption of the most impracticable and erratic notions on the most inadequate basis of observation and fact, and at the same time inflates with a lofty and unapproachable egotism which precludes its possessor from meeting the views of an opponent with anything but epithets, however superior his opponent may be to himself in learning, experience, or sagacity. A precipitancy that has no nerves left for investigation and patience at criticism marks his every act and word. Laughlin shows this demoralizing precipitancy, so fatal to level-headed usefulness, by speaking of the silver coin, whose equal dignity with gold coin in all legal respects is irrevocably fixed in the letter of the Constitution of the United States, “as the false silver dollar,” thereby implying, of course that from 1853 to 1870, when silver happened to be worth more than gold, we must have been under a “false gold dollar.”

Laughlin also calls those navigation laws which have never existed either among barbarous or medieval nations, but which began in England under Cromwell, “barbarous and medieval.” He might as well call steam or the art of printing “barbarous and medieval.” Sensible man weary of these impudent epithets flung at them by young and graceless upstarts who have still their spurs to win in everything that distinguishes useful men from snobs.

SourceThe Inter Ocean (Chicago, Illinois) April 15, 1885, p. 4.

Image Source: Portrait (1885-88) of James Lawrence Laughlin. Harvard Art Museums/Fogg Museum, Transfer from the Carpenter Center for the Visual Arts.

Categories
Chicago Exam Questions

Chicago. First price theory course exams. Hanoch, 1964

Giora Hanoch graduated with a doctorate in economics from the University of Chicago in December 1965 with his dissertation “Personal earnings and investment in schooling.” He held the rank of assistant professor of economics for the academic year 1964-65, after which, according to his entry in the AEA 1969 Biographical Listing of Members,  he returned to Hebrew University, Jerusalem, in 1965. With many visiting appointments throughout his career, his academic home was Hebrew University.

Clearly the faculty thought highly enough of him in his fourth year at Chicago to entrust him with the first quarter of the 300-level price theory sequence (Autumn Quarter, 1964).

_________________________

Giora Hanoch, Professor Emeritus of Economics
Hebrew University of Jerusalem

1932. Born in Haifa, Israel
1960. A.B. Hebrew University.
1961. A.M. Hebrew University.
1965. Ph.D. University of Chicago. Thesis “Personal Earnings and Investment in Schooling”
1970. Visiting Lecturer, Harvard University.
1974. Visiting Lecturer, Harvard University.
1975. Visiting Lecturer, University of California, Los Angeles
1975— Fellow of the Econometric Society.

Source: “Giora Hanoch, economist”, Prabook website.

_________________________

Economics 300
G. Hanoch

Mid-Term Examination
November 18, 1964

I. (60 points)

Answer the following True, False, or Uncertain. Explain your answer briefly.

  1. If two individuals engage in barter, or direct exchange of goods, then always either: a) One individual benefits by the transaction while the other one is hurt; or b) Both are neither benefited nor hurt.
  2. In a perfect market economy, each consumer participates equally in determining what is produced.
  3. If an increase in the demand for X results in an increase i n the price of X, the demand for X is upward sloping.
  4. If the demand for X has unitary elasticity (η = -1), changes in the price of X will not affect the total expenditures on all other goods.
  5. If one good is inferior, at least one other good purchased by the consumer has to be income-elastic (ηxI> 1).
  6. If the marginal revenue is decreasing with an increase in the quantity X, the demand for X is inelastic.
  7. The substitution effect of a decrease in price, as defined by Slutsky, is positive for a normal good and negative for an inferior good.
  8. If the market for beef is in a stable equilibrium, changes in the supply of beef will have little or no effect on its price.
  9. It is possible for a consumer to buy a fixed positive) quantity of X every month, whatever the price of X may be. (i.e., his demand for X has zero elasticity for all prices).
  10. The demand for agricultural products is inelastic; hence plentiful harvests result in lower incomes for farmers, in a free market economy.
  11. In view of (10), each individual farmer can improve his own position by destroying a part of his production in good years.
  12. A linear and downward-sloping demand curve is always elastic at high prices and inelastic at low prices.
  13. If the Laspeyres quantity index between two periods is 1.10 and the Paasche index is 0.90, the consumers’ taste must have changed,
  14. The cross-elasticity of demand for left shoes with respect to the price of right shoes is zero.
  15. A consumer with a utility function is in equilibrium if the marginal utility of each good is proportional to its price.
  16. If all prices increase by 10%, but money income remains the same, the quantity of each good purchased will decrease.
  17. The demand of a consumer for X cannot be infinitely elastic at every quantity of X, because of the budget constraint.
  18. In an economy where the king distributes all the goods and services as free gifts to the consumers, all the prices are zero. Hence there is no place for price theory in that country.
  19. The demand for X is of unitary elasticity, and 200 similar firms sell X. A reduction of 1% in the price PX charged by one firm will result in doubling that firm’s sales, if other firms sell the same quantity at any price.
  20. Because of transportation costs, prices will differ in different geographical locations, whether or not there exists free competition in the market.

II. (40 points)

Two consumers, A and B, have equal and stable tastes and incomes. In December, each spent his entire monthly income on x units of X and y units of Y, when the prices in the market were $2.00 for X and $5 for Y. Consumer A accepted an offer of his employer to be paid in kind, by receiving the same quantities y and y every month directly. (He could still exchange any quantity of X and Y at the market, for the current market prices). B’s money income remained the same.

The following prices prevailed in the market during the next few months:

Month

$ per unit of X $ per unit of Y
1 2.00 5.00
2 2.20 5.50
3 2.00 5.50
4 2.00 4.50
5 1.80 4.50
6 2.20 4.50

1) Compare consumer A’s position in each of these months with his position in December (was he better-off, worse-off, or indifferent?)

2) Compare the positions of A and B in each month.

NOTE: Use budget lines (and, if necessary, indifference curves) for your analysis. Do not attempt to answer more questions than you were asked. Be brief and clear.

_________________________

ECONOMICS 300
G. Hanoch

FINAL EXAMINATION
December 14, 1964

(two hours)

I. (40 points)

Mark the following True, False, or Uncertain. Explain your answers very briefly.

  1. A monopolist can afford to pay wages below the market wage rates.
  2. A rise in the price of gasoline will lead to a rise in the price of tires.
  3. In a long run competitive equilibrium, the marginal firms produce where marginal costs equal average total costs.
  4. If a firm is in long run equilibrium, it is also in short-run equilibrium, whether it is a competitive or a monopolistic firm.
  5. If a production function is characterized by constant returns to scale, an increase in the use of one factor by 10% will increase output by less than 10%.
  6. A rise in the price of any factor used by the firm (other things unchanged) will always lead to a decrease in production by the firm.
  7. A firm producing the same product in many plants will determine the quantity produced in each plant so that average costs will be equal in all the plants.
  8. If a firm has zero variable costs, then its best profit output is where the elasticity of demand for the product is unitary.
  9. A firm will carry production to the point where the marginal productivities of all variable factors are equal.
  10. In a competitive industry with external economies, the total short run supply curve of the industry shifts to the left when there is a permanent decrease in demand for the product.

Il. (30 points)

A monopolist is faced with the following stable demand schedule for his patented machines:

Price per machine
(thousand dollars)

Quantity
per month
TR MR TC MC
40 1
35 2
30 3
25 4
20 5
15 6
10 8
5 10

The Costs of production are $5000 per machine, and the fixed costs are $16000 per month.

1.) Compute total and marginal revenue and total and marginal costs in the table above.

2.) Find the equilibrium price, quantity and profits of this firm.

3.) A tax of $60,000 per month is imposed on the firm. Find the new price, quantity and profits.

4.) Instead, a tax of 60% of the market price is imposed on the machines. What will be the monopolist price, output, profits? The tax revenues?

5.) Alternatively, a tax of $24000 per machine is levied. What are the equilibrium price, quantity, profits and tax revenues? What will be the long-run equilibrium quantity?

6.) If no tax is imposed, but a maximum price of $10,000 is enforced, what will be the quantity sold? The Profits?

7.) State your preference among the 5 alternatives ((2) – (6)) above, and justify your choice briefly.

III (30 points)

The current charge for telephone service in city C is $6.40 per month, allowing the consumer 80 free local calls every month, Each additional call costs five cents. Installation is free, and no long-distance calls are available.

NOTE: In the following, assume that each consumer behaves rationally, has constant money income and tastes, with convex indifference curves and no saturation in the relevant range.

Use separate diagrams for each sub-problem. Be precise.

  1. Use a diagram with money-income Y and phone calls X on the axes, to show a consumer’s budget constraint. Be careful to show all the combinations of X and Y available to him, including the case where no service is installed.
    (This portion is crucial for the rest of the problem).
  2. Use indifference curves between Y and X to analyze the consumer’s decision whether to have a telephone installed or not.
  3. Consumer A chooses to have a telephone, and he uses 120 calls every month. Show his equilibrium position geometrically. What is the average price (in cents) of a phone call for him? What is his marginal rate of substitution between money and phone calls?
  4. If the current rates are replaced by a flat rate of 7 cents a call for any number of calls,

(a) Show consumer A’s new budget line, compared with the current position.
(b) Would he now use more or less than 120 calls per month?
(c) Would he be better-off, indifferent, or worse-off relative to the current position?

  1. Consumer A claims that he would prefer to pay a flat rate of 84 per call rather than the current rates. Could he be rational? (demonstrate your answer geometrically).
  2. Consumer B uses only the 80 “free” calls every month, given the current rates. Compare (as in (4)) his consumption and welfare positions with the alternative of being charged a flat rate of 8¢ per call for any number of calls. Could he be indifferent with respect to the two alternative rates?

 

Source: Harvard University Archives. Papers of Zvi Griliches. Box 130, Folder “Syllabi and exams, 1961-1969”.

Image Source: Giora Hanoch in “These Israelis Were Present at the Declaration of Independence.”  Haaretz. Apr. 17, 2018

Categories
Economists Gender

UK. Chicago newspaper article, Fawcett and His Wife, 1872

 

A newspaper account from 1872 provided me by serendipity. A Chicago reader would have learned that Millicent Garrett Fawcett at age 25 was considered “the best speaker of any of the women who have come into public life”, at least in her own country, and “much more than ordinarily pretty”. Her Political Economy for Beginners ran through ten editions over forty years (Tenth edition, 1911).

__________________________

Link to Millicent Garrett Fawcett’s Autobiography:  What I Remember (1925)

__________________________

Fawcett and His Wife

Prof. Fawcett, the liberal member of Parliament, who came so near overthrowing Mr. Gladstone is blind. When a pretty well-grown boy, but before entering the university, an accident destroyed one eye, and the spreading inflammation soon took the other. As soon as his health was restored he continued his studies with an attendant who acted as guide, amanuensis and reader. High honors and finally a fellowship at Trinity College, Cambridge and subsequently the publication of a work on political economy, secured him a professorship in this same college. Other publications on “Pauperism,” “Land Tenure,” and the various questions that English radicals are airing, won him great favor among the working classes, and in 1865 he entered Parliament as the representative for Brighton, a constituency composed chiefly of trades-people. Prof. Fawcett follows in the line of Mill, but as he is far less subtle, he has the good fortune to be much more popular with the ordinary mind. He is honest and has a steady nerve. He is now 38, just in the prime of his powers, with a markedly strong physique, as opposed to fineness of fibers and nervous receptivity. On the evening of the day that the telegram announced the death of President Lincoln, Prof. Fawcett was in a social gathering of liberals, and heard from a girl of 18 the exclamation, “It would have been less loss to the world if every crowned head in Europe had fallen.” He asked to be introduced to this spirited girl, who has been Mrs. Fawcett for the last five years. Mrs. Fawcett is now 25, and is, with the exception of her sister, Mrs. Anderson, perhaps the most popular woman in England. She is the best speaker of any of the women who have come into public life. She is the author of a political economy adapted for use in girls’ schools, and appears again as the largest contributor in a volume of essays, by Mr. and Mrs. Fawcett just published. She has the same clear, logical, practical type of mind as Prof. Fawcett, with an added feminine fineness. It would be difficult to find two people more consonant in their tastes and aims. Mrs. Fawcett is slight in figure and much more than ordinarily pretty; is neither distrustful nor presuming, and has that perfect balance of mind that enables her to use all her power. Her sister, Mrs. Garrett Anderson, was the first regular woman physician in that country. She is a member of the London School Board, and, what no one fails to add dresses extremely well. She has the reputation of being remarkably skillful in her profession, but I am satisfied that her exceptional qualities, like those of Mrs. Fawcett, lie in the line of practical effectiveness, rather than in original thought. The social popularity of these sisters illustrates a contrast between English and American society. Americans do not like peculiar people, not even people of peculiar excellence. A domestic uniformity is the aristocratic standard, and women who step out of this suffer more than men do. Not so there. If one shows intellectual powers above other women, or superior practical efficiency in public affairs, just so much is added to her social rank. Women are dealt with in this just as fairly as men are. Intellectual merit is the one coin that in England gets everything in exchange. It is rather singular that these three sisters [unnamed third sister: Agnes Garrett, interior designer and suffragist] should all have distinguished themselves in strong-minded lines, since the mother holds the most conservative views in regard to women’s work, and the father has no interest beyond personal pride in the success of his daughters.

Source: Chicago Evening Post, June 1, 1872, p. 3.

Image Source: National Portrait Gallery, “Henry Fawcett; Dame Millicent Garrett Fawcett (née Garrett“.

Categories
Berkeley Economists Gender Oxford Radcliffe Smith

Radcliffe/Oxford. An economics major who got away (to history of art). 1919

Ruth Doggett was on the start of a promising academic career as an economist until she completely switched her focus to Italian art history, having (presumably happily) worked together with her art historian husband, Clarence Kennedy, in Florence. As the record shows, economics’ loss turned out to be art history’s gain. 

Something in me hopes that I find a case of an art historian who turns to economics. What are the odds?

_____________________________

Ruth Wedgewood Doggett Kennedy
c.v.

Ruth Wedgewood Doggett was born August 19, 1896 in Greenville, Rhode Island. Her father was the President of Springfield College.

Ruth Doggett began her undergraduate studies 1915-16 at the University of California, Berkeley. In 1919  she graduated with an A.B. (Phi Beta Kappa) from Radcliffe College, receiving a magna cum laude in Economics.

Her obituaries report that she was an instructor in Economics at Smith College 1919–20, but I have not been able to confirm any economics and sociology course for her other than “Principles of Sociology” (e.g. Smith Catalogue 1921-22, p. 69).

Ruth Doggett spent a year (1921) at Lady Margaret Hall, University of Oxford where she was awarded a Diploma in Economics with first class honors. In London she married the young art historian/photographer Clarence Kennedy in 1921. They had previously met at Smith College.

The young couple returned to Smith College, Ruth Doggett Kennedy  as an instructor in Economics (1921–23) and Clarence was appointed assistant professor of art history.

In 1923 the Kennedys moved to Florence to teach in the Art Department’s Division of Graduate Study program. Ruth Kennedy served as assistant to the Director of Graduate Study in Art, 1925–26, 1927–28. She was a special lecturer in History of Italian Art, 1928–29, Smith College.

Ruth Kennedy was awarded a John Simon Guggenheim Foundation fellowship to complete a study of the Florentine painter, Alesso Baldovinetti and of his associates, in Italy; tenure, six months from March 10, 1930.

From that time on she lectured on art at Smith, Springfield College and Wellesley. Her major publications were:

  • Alesso Baldovinetti, a critical & historical study. New Haven: Yale University Press, 1938.
  • The Renaissance painter’s garden. New York: Oxford University Press, 1948.
  • The Italian Renaissance. New York: Art Treasures of the World, 1954.

1960-61. Ruth and Clarence Kennedy were invited to serve as Resident Art Historians at the American Academy in Rome.

1961 Ruth Kennedy officially retires as emeritus professor of art at Smith College..

  • Novelty and tradition in Titan’s art. Katharine Asher Engel lectures. Northampton, Mass., Smith College, 1963.

Ruth Wedgewood Kennedy died November 30, 1968 in Boston.

_____________________________

Obituaries.

Lee, R. W. (1969). Ruth Wedgwood Kennedy. Renaissance Quarterly, 22(2), 206–208.

The Boston Globe, December 1, 1968, p. 110.

_____________________________

From the Finding Aid for Kennedy Family papers, Smith College Archives.

Clarence Kennedy was born in Philadelphia in 1892. He received his bachelor’s degree in architecture and a master’s degree in art history from the University of Pennsylvania, and studied at the American School of Classical Studies in Athens, Greece, as a Charles Eliot Norton Fellow of Harvard University. While working on his Ph.D. at Harvard, he joined the Smith College Art Department faculty. Kennedy received his doctorate from Harvard in 1924. His dissertation was titled The Effect of Lighting on Greek Sculpture.

Ruth Wedgewood Doggett was born in Greenville, Rhode Island in 1896, and was raised in Springfield, MA, where her father was President of Springfield College. She attended the University of California at Berkeley for two years and completed her undergraduate education at Radcliffe College with a degree in Economics, magna cum laude. She taught at Smith College in the Economics Department for a year after her graduation and then spent a year at Lady Margaret Hall of Oxford University furthering her study of economics.

Clarence Kennedy and Ruth Wedgewood (Doggett) Kennedy were married in England in 1921. At the time Clarence was traveling in Italy and Greece, photographing classical sculpture. The next fall they returned to Northampton, where both had held positions in the Art and Economics departments, respectively. In 1923 the Kennedys moved to Florence to teach in the Art Department’s Division of Graduate Study program. By this time Ruth had begun to establish herself as a Renaissance scholar, while Clarence continued his photographic and academic work. Their collaboration was continuous and they were among the pioneers of modern techniques in the study of art history. Among their innovations was the teaching of art history in situ instead of in the classroom. During this time the Kennedys had two children, Melinda, born in 1924, and Robert, called Bobby, born in 1928.

Ruth Kennedy was a member of the Art Department from 1941 to 1961. She taught courses on Italian Renaissance artists and on the cultures and cities that informed their art. During her time at Smith she undertook research on Alesso Baldovinetti, Fra Bartolomeo, Francesco Laurana, as well as projects on flowers in Renaissance art. She also was active nationally and internationally in her field of Renaissance art; her articles and reviews appeared often in art journals and she served on the editorial board of Art in America, Renaissance Quarterly, and the Art Bulletin. During her career Ruth received fellowships from the Guggenheim Foundation and the American Philosophical Society. She also lectured at other institutions, including Springfield College and Wellesley College.

From 1928 to 1932, Clarence Kennedy and Smith College published Studies in the History and Criticism of Sculpture, a seven-part series of volumes, issued in editions of 100, containing over three hundred black and white gelatin photographs of Ancient and Renaissance sculpture.

After Clarence and Ruth’s return to Northampton from Italy in 1933, Clarence continued to teach art history and photography; he soon added typography to his courses when he and Ruth set up the Cantina Press in their home at 44 Pomeroy Terrace in 1936-37. Cantina Press published little under its own imprint, but the Kennedys helped to establish the tradition of typography and printing at Smith College and produced much ephemeral work, such as invitations, broadsides, and programs.

Clarence Kennedy collaborated with scientist and inventor Edwin H. Land, co-founder of the Polaroid Corporation, on a system for the projection of stereoscopic lantern slides using Land’s invention of Polaroid filters over the lenses of a double projector. Viewing the projected images through special glasses with Polaroid filters identical to those on the projectors, the audience could see the image in three dimensions. Clarence also worked with Land during World War II on development of a Vectograph system using polarized stereographic images for three-dimensional maps. He was a member of the Monuments and Fine Arts Commission, established by the United States government to minimize the destruction of works of art within enemy-held territory during World War II. Clarence was also a consultant to the Eastman Kodak Company on photographic matters.

Together, Ruth and Clarence were invited to serve as Resident Art Historians at the American Academy in Rome for 1960-61.

Ruth Kennedy became a Professor Emeritus at Smith in 1961, but continued to lecture, research, and work on potential publications until her sudden death after a short illness on November 30, 1968 in Boston.

Clarence Kennedy retired from Smith in 1960, and died on July 29, 1972 in Northampton.

Melinda Kennedy (1924-2002) was the first child and only daughter of Clarence and Ruth Kennedy. Melinda attended Smith College and graduated with the class of 1945. Melinda was married for several years to Alfred Lester Talkington, who was known as Hank. Melinda and Hank had two daughters, Sylvia and Amy. Hank also had a daughter, Jo Lynn, from a previous marriage. Melinda was a poet and translator, and taught English for many years at Choate Rosemary Hall in Wallingford, CT.

Image Source:  The Radcliffe College Yearbook 1919, p. 31.

Categories
Berkeley Chicago Economics Programs Economists

Chicago. The Education of Zvi Griliches. Through Ph.D. 1957

 

The two documents transcribed for this post provide wonderful detail about the economics training received by Zvi Griliches whose academic career passed from Hebrew University, through the University of California, Berkeley, and ultimately through the University of Chicago to Harvard.

Griliches was responsible for graduate admissions in the Harvard economics department back when I was applying to graduate school (1974). When I went to Cambridge to visit the Harvard and M.I.T. departments, I pressed Griliches (the only professor at Harvard with whom I could get an appointment) for him to tell me what in his opinion the difference between Harvard and M.I.T. was. He smiled (hopefully amused by my naive presumption) and replied that M.I.T. provided more of a “bootcamp training” than Harvard would. He did make that sound like a bad thing. In any event, M.I.T. was better at recruiting, able on short notice to line up appointments to talk with Evsey Domar and Charles Kindlberger plus a handful of graduate students. Still I have to admit that Griliches did warn me what I was getting myself into.

Zvi Griliches was awarded a Social Science Research Council Research Training Fellowship in 1955-56, and from information in the supplementary statements below, it is clear that the application was written sometime in the early months of 1955 (Chicago’s Winter Quarter 1955). So while it is possible that he was applying for more than this single fellowship, there is no indication of any other fellowship at that time being considered in Griliches’ papers in the files at the Harvard Archive that I consulted.

Questions for the Price Theory prelim exam for the Winter Quarter 1955 have been posted earlier. From Milton Friedman’s papers, we know that Griliches got the top grade (by a long shot) on that particular exam.

Griliches received a two year appointment at Chicago beginning Oct 1, 1956— “to give service for the National Science Foundation Econometric Model Research Project on a ninety per cent time basis and for the Department of Economics on a ten per cent time basis with total salary of $5,000 per annum”. So it was certainly reasonable for him at the start of the second year of his contract to put his academic record on file with the University of Chicago Vocational Service and Employment Office. That is the second document transcribed below.

Economic in the Rear-view Mirror’s “Believe it or not!”

Graduates listing themselves with the University of Chicago’s Vocational Service and Employment Office were asked even as late as the Autumn Quarter of 1957:

Any racial or religious institutions in which you would prefer to teach?
Any racial or religious institutions in which you would prefer not to teach?

Easy to believe, and the documentary record indeed shows, that Zvi Griliches answered “No” to both questions.

___________________________________

Supplementary Statements from Winter Quarter 1955 in Griliches’ fellowship application for 1955-56

 Zvi Griliches

SUPPLEMENTARY STATEMENT “C”:

For attainment of the objectives outlined above I think that the following knowledge and training is desirable:  1) economic theory including mathematical economics; 2) statistics and econometrics including all the modern developments and also experience with efficient computational procedures; 3) agricultural economics; and 4) some knowledge of historical methods.

  1. Economic theory and mathematical economics will be very important in my future work because they provide the framework for the actual quantitative work. They suggest which are the important variables in different problems and indicate something about the form of their interrelationships. They also provide a check on the internal consistency of our models and are the source of most of our hypotheses. I believe that I have a good knowledge of basic economic theory and a reasonable familiarity with mathematical economics. My major graduate courses in this field were:

R.G. Bressler Jr. — Production Economics — In this course I was introduced to the pure theory of production and to the interrelationships of cost and supply curves.

R.G. Bressler Jr. — Seminar in Agricultural Marketing Organization — This course, in spite of its name, dealt primarily with problems of cost measurement, location theory, and general equilibrium.

Robert Dorfman — Advanced Economic Theory A-B1 — This was the major graduate course in Economic Theory at the University of California, covering Price Theory, Distribution Theory, and introducing us to Income and Employment Theory.

Robert Dorfman — Mathematical Methods in Economics — This was my introduction the Mathematical Economics proper. It dealt with general maximization problems, the pure theory of consumers’ choice, and in particular with dynamic difference equations models. The last topic will be very important in the construction of my model.

A.C. Harberger — Price Theory A — Covered more advanced topics in price theory and problems of definition and measurement of utility.

D. Gale Johnson — Price Theory B3 — This course covers distribution theory and related topics.

            I have also taken in the past and intend to take in the future a series of courses in Monetary and Fiscal Theory which I shall not list here.

            I also intend to participate in the Seminar in Mathematical Economics to be given in the spring quarter of 1955 by G. Debreu at the University of Chicago. In spite of all the above, I shall still lack adequate knowledge of Mathematical Economics. I need especially a better knowledge of growth models and of stochastic difference equations. I think, however, that I shall be able to acquire this necessary knowledge through individual study, as my work progresses.

            I am aided in my knowledge of mathematical economics and also of statistics and econometrics by a good undergraduate training in calculus and an individually acquired knowledge of matrix and vector algebra. Nevertheless, this is not enough. As it forms a basis for most of the other fields, I should learn more mathematics. I intend to do so after I have completed the preliminary Ph.D requirements both through intensive studying on my own and also by auditing some courses at the university.

 

  1. A good knowledge of statistics and econometrics is indispensable for quantitative work in agricultural economics. Though this is a field where there is always more to learn, nevertheless, I think that I have a basic knowledge of the most important techniques. My major courses in this field were:

George Kuznets — Analytical Methods A — This was my introduction to the theory and methods of multiple regression, weighted regression, testing hypotheses, and non-parametric tests. Within the framework of this course I wrote a paper “Demand for Clingstone Peaches on the Grower Level” which introduced me to modern computational procedures and the use of modern computational equipment.

Ivan Lee — Analytical Methods B — In this course I was introduced to simultaneous equations, the identification problem, maximum likelihood estimates, analysis of variance, and sampling theory. Within the framework of this course I wrote a paper “Clingstone Peaches: Demand and Supply Relationships on the Grower Level” applying both least squares and limited information techniques.

Roy Radner — Statistical Problems of Model Construction1 — Introduced me to decision theory, covered in greater detail the Markov Theorem and maximum likelihood estimates.

Martin Beckman — Allocation of Resources in Production3 — This course is introducing me to the valuable new technique of activity analysis (linear programing).

W.H. Kruskal —  Mathematical Statistics I2 — The principal topics of this course are: point and set estimation; hypothesis testing; elements of multivariate analysis; elements of linear hypothesis theory; typical nonparametric procedures.

            In the addition to the above I profited greatly from work with Professors Varden Fuller and Ivan Lee (Summer 1953), which made me familiar with census data, BAE publications, and other major sources of data in agriculture: and from my work with Professor Sidney Hoos (Summer 1954), which provided practical experience in the application of modern econometric techniques. I also have participated and shall continue to participate in the Seminar in Econometrics conducted by members of the Cowles Commission at the University of Chicago.

            All this of course is not enough. I shall have to learn much more. Some of it I shall still get at the university, but the greater part I shall have to learn on my own as my work progresses.

 

  1. A thorough knowledge of agricultural economics is important as it will provide both the framework and background of my work. I believe that I possess a reasonably good knowledge of this field. I have received both the B.S. and M.S. degrees in agricultural economics and have read widely in the field. Some of my courses in this field were:

George Mehren — Seminar in Agricultural Marketing — Introduced me to the practical and theoretical problems arising in the administration of agricultural marketing and adjustment programs.

Murray Benedict — Agricultural Production Economics — Dealt with the theoretical issues underlying policy problems in agriculture.

Varden Fuller — Seminar in Agricultural Policy1— Dealt with current policy issues and their economic implications.

C.M. Hardin — Seminar in Agricultural Policy2 — This course is introducing me to the consideration of current agricultural policy issues from the point of view of Political Science.

T.W. Schultz — Choice and Possibilities in Economic Organization — Dealt primarily with economic development and its impact on agriculture.

D. Gale Johnson — Incomes Welfare, and Policy3 — This course is introducing me to more advanced topics in agricultural economics and policy.

            I have gained also from participation in departmental meetings and seminars, both at the University of California and at the University of Chicago. Three years of my life spent working on farms (1947-50) and a summer (1952) as a research assistant with the California Packing Corporation collecting yield data have enriched my understanding of agriculture and its problems.

 

  1. As time series are used to a great extent in quantitative work, some knowledge of historical methods is quite important. I am fortunate in this respect to have had a very intensive and profitable year of undergraduate study in History at the Hebrew University, and in particular a course in “Introduction to historical literature and methods” by Professor Richard Koebner

            The only way one really becomes adept in quantitative work is by doing quantitative work. In a sense, this is the purpose of my project. As a result of work on my project I should gain experience and facility in using both theory and quantitative methods.

I have a good knowledge of Russian, German and Hebrew.

—————————————–

1 I audited this course
2 I am auditing this course
3  am currently taking this course

 

SUPPLEMENTARY STATEMENT “D”:

I shall have completed all the required course work by June 1955. I intend to take the Ph.D. preliminary examination in Winter 1955, and the preliminary examinations in Money and Banking, and Agricultural Economics in Summer 1955. I have already taken and passed a reading examination in German, and I shall take the examination in Russian in February of 1955. Hence, I hope to have completed all the requirements toward the Ph.D. degree, except the dissertation and final oral examination, by August 1955, and before the fellowship goes into effect.

The preliminary title of my thesis is “A study of the factors determining the development, distribution, and acceptance of new technology”.

The faculty adviser is Professor D. Gale Johnson,

 

SUPPLEMENTARY STATEMENT “E” :

1950-51. A student at the Hebrew University, Jerusalem.

1951-54. Student at the University of California, Berkeley.

Summer 1952. Research Assistant with the California Packing Corporation. Collection of yield data. $1.10 per hour.

Fall 1952. Awarded the D. Solis Cohen Scholarship. This scholarship was awarded to me during the following two semesters.

May 1953. Election to Phi Beta Kappa.

June 1953. Awarded the degree of Bachelor of Science with highest honors in Agriculture.

Summer 1953. Research Assistant with the John Haynes Foundation, working under the direction of Prof. Varden Fuller, at the University of California. Salary: $325/month

1953-54. Jesse D. Carr Fellow in Agriculture at the University of California.

Summer 1954. Research Assistant at the Gianini Foundation of Agricultural Economics, University of California; working under the direction of Professor Sidney Hoos. Salary —$290 a month.

September 1954. Awarded a Master of Science degree in Agricultural Economics by the University of California.

1954-55. A University Fellow and full time student at the University of Chicago.

 

SUPPLEMENTARY STATEMENT “F”:

  1. “Demand for Clingstone Peaches on the Grower Level”, Berkeley,  January 1954, Typewritten manuscript,
  2. “Clingstone Peaches for Canning: Demand and Supply Interrelationships on the Grower Level”, Berkeley, June 1954,
  3. “The Differential Spread of Hybrid Corn: A Research Proposal”, Chicago, December 1954, pp. 1-20.

All three papers are available on loan from me. All are unpublished typewritten manuscripts.

 

Source: Harvard University Archives. Papers of Zvi Griliches, Box 129, Folder: “Correspondence, 1954-1959.”

_______________________________

THE UNIVERSITY OF CHICAGO
VOCATIONAL GUIDANCE AND PLACEMENT

EDUCATIONAL REGISTRATION FORM

Date: September 30, 1957
Name in Full: Hirsch Zvi Griliches
Current Address: 6011 Kimbark, Chicago 37, Ill.
Telephone: Bu 8-1975
Permanent Address: ditto

 

PERSONAL DATA

Date of birth: 9/12/1930. Place: Kaunas, Lithuania
Are you a U.S. citizen? No
If through naturalization give date. If not, explain status: Permanent resident (immigrant), expect. naturalizt. in 2 yrs.
Height: 5’11
Weight: 160
Marital status: Married
Number and ages of children: 1 daughter, 9 months.
Are you a veteran? Of the Israeli Army.
Physical handicaps: None
Church (if you wish to indicate): Jewish
Scholastic honors: S.B. with Highest Honors in Agriculture (U of Calif., 1953), Phi Beta Kappa
Scholarships (give dates and schools): Solis D. Cohen Scholarship, Univ. of Calif., 1952-53
Fellowships (give dates and schools): Jesse D. Carr (Univ of Calif., 1953-54), University (U of Chicago, 1954-55), Social Science Res. Council Research Training Fell. 55-56
Certificates held: None

 

EDUCATIONAL AND RELATED EXPERIENCE

List chronologically all work experience (including teaching, government, business, practice teaching, and experience in armed services)

June 1953 to Sept 1953. John Hanes Foundation, Berkeley, Calif., Research Assistant

June 1954 to Sept. 1954. Univ. of Calif., Berkeley, Research Assistant, Price Analysis.

Oct. 1954 to Sept 1955. Office of Agricult. Economic Research, Chicago, Research Assistant.

Oct. 1956 to date, U of Chicago, Assistant. Prof., Ag. Economics, Gen. Econ. Theory.

 

ACADEMIC AND PROFESSIONAL TRAINING

(If this space is insufficient, attach another sheet)

UNDERGRADUATE COURSES: (List title, not catalogue number, and follow with the number of semester hours; e.g. Shakespeare, 3. One full course in the College of the University of Chicago equals 3 semester hours.)

First Year

Second Year Third Year

Fourth Year

Hebrew 10 Geology 6 Botany 3
English 8 Introd. Econ 6 Calculus 6
Latin 8 Intern. Trade 3 Ag Econ Theory 6
Russian 4 Statistics 3 Ag. Marketing 3
Westr. Civil. 6 Agric Policy 3 Ag Policy 3
Polit. Theories 8 Range Mangmnt 3 Hist. of Ec. Thght 3
Medieval History 8 Zoology 3 Irrigation Econ. 3
Sociology 8 Agronomy 3
Intnat.Econ. 3

 

GRADUATE COURSES AT THE UNIVERSITY OF CHICAGO (List Course Title.) (One full course in the Divisions of the University of Chicago equals 3½ semester hours)

Instructor

Title of Course Instructor

Title of Course

Harberger Price Theory A Hamilton Banking and Monetary Policy
Monetary and Fiscal Policy Metzler Monetary Asp. of Inter’l Trade
Recent Dev. in Economics Beckman Alloc’n of Res. in Prod.
Schultz Choice & Possib. in Econ. Org. Audited:
Econ. Org. for Stability Savage Introd. To Probability Theo.
Regression & Anal. of Varian.
Johnson Price Theory B Theil Math. Economics
Income, Welfare, & Policy Radner Econometrics
Friedman Price Theory A & B
Tolley Money

 

GRADUATE COURSES TAKEN ELSEWHERE (University of California, Berkeley)

Instructor

Title of Course Instructor

Title of Course

Clark Agric Marketing 3 Kuznets Analytical Methods A 3
Mehren Agric Marketing. Sem. 3 Lee Analytical Methods B 3
Bressler Ag Market Organ. Sem. 3 Dorfman Math Methods of Econ 3.
Bressler Ag Production Theory A 3 Audited:
Benedict Ag Production Theory B 3 Dorfman Econ Theory A & B 6

 

SUMMARY OF ACADEMIC AND PROFESSIONAL TRAINING AS OF Oct. 1, 1957

MAJORS SEMESTER HOURS MINORS
(OR RELATED FIELDS)

SEMESTER HOURS

Undergraduate

Agric Econ 15 History 22
Economics 15 Math and Statistics

9

Graduate

Agric Econ 24 Econometrics & Stat 6 + 9 aud.
Econ Theory & Math Econ 15 +15 aud Money

12

Thesis field and preliminary fields: Agricultural Economics, Economic Theory, Monetary Theory.
Education Courses:  None

 

ACADEMIC AND PROFESSIONAL TRAINING

List all schools attended. Begin with high school from which you graduated. Include work in progress at the University of Chicago and Foreign [Universities]

Dates of Attendance

Institutions—Location Major Subject Minor Subject

Degree and Date Awarded

6/50 Dept. of Education, State of Israel External Matriculation exams passed 1950
9/50 to 6/51 Hebrew University, Jerusalem History Sociology
10/51 to 6/54 University of California, Berkeley Agric. Econ Agric. Market. S.B. 1953
S.M. 1954
10/54 to 8/57 University of Chicago, Chicago Economics Agric. Econ. A.M. 1955
Ph.D. 1957

Title of Master’s thesis: no thesis

Title of Doctor’s thesis: Hybrid Corn: An Exploration in Economics of Technological Change.
Thesis adviser: T.W. Schultz

 

EXTRA-CURRICULAR ACTIVITIES

Single check activities in which you have participated. Double check those which you can direct [coach/play].

Assemblies, Athletics, Audio-Visual, Band, Camping, Chorus, Civic Organizations, Crafts, Curriculum Planning, Debate, Dramatics, Gymnasium Activities, Orchestra, Parent-Teachers activities, Piano, Playground, Public Addresses, Pupil Participation in Government, Reading, Rhythms-Dances, School Clubs, School Publications, School Publicity, Speech, Vocational Guidance.

[Note:  Only School Clubs was checked (single checked) from the list. It was the Political Economy Club in college]

What foreign languages do you speak? Hebrew, Russian, German, Lithuanian, Yiddish.

Can you type? Poorly. Take dictation? No. Bookkeeping knowledge? No.

 

ADDITIONAL INFORMATION

(For our use only—not included in credentials mailed to employers.)

PUBLICATIONS:

“Specification bias in estimates of production functions,” Journal of Farm Economics, February 1957.

“Hybrid Corn: An Exploration in the Economics of Technological Change,”Econometrica, October 1957.

Book reviews in the Journal of Political Economy

MEMBERSHIPS:

American Economic Association
American Farm Economics Association
Econometric Society
Fellow of Royal Economic Society

 

REFERENCES
Instructors at the University of Chicago

List at least two University of Chicago instructors who are able to evaluate your course work.

T.W. Shultz
A.C. Harberger
D.G. Johnson
Carl Christ

 

INSTRUCTORS AT OTHER COLLEGIATE INSTITUTIONS

List instructors at other schools from whom you would like to have letters of recommendation.

R.G. Bressler. Dept. of Agric. Economics, Univ. of Calif., Berkeley.
Sidney Hoos.  Ditto.

 

ADMINISTRATIVE OFFICERS

If you have had teaching or administrative experience, list administrative officers who can report on your work (one for each position you have held).

D.G. Johnson, Univ. of Chicago. Act. Chairman
H.G. Lewis, Univ. of Chicago. Director of Research Center.

 

At what other university or college placement office are your letters of recommendation on file?   None.

 

OCCUPATIONAL CHOICES

List three position choices. Be very specific as to (1) courses you can teach within your own department (e.g., if Sociology-Social Psychology, Marriage and Family, Theory); (2) kinds of institutions (University, Liberal Arts College, State Teachers College, Junior College, High School, Junior High School, or grades); (3) other types of positions (Registrar Dean, Superintendent, Business Manager, Critic, Supervisor, etc.).

University, Land Grant or Liberal Arts College, teaching position with opportunities for research. Economic Theory, Agricultural Econ., Econometrics, Money.

Date available (month and year) September 1, 1958
Locality preferred East or West Coast.
Are you limited to that area? No.
Would you apply for positions in foreign countries? Yes
Any racial or religious institutions in which you would prefer to teach? No
Any racial or religious institutions in which you would prefer not to teach? No
Present or last salary $6500 (Confidential) for 11 months.
Minimum salary you would consider $7000 (Confidential) for 11 months.

Your registration is incomplete without six photographs (not larger than 2 ½ by 3 ½ inches).
Pictures are important.

Source: Harvard University Archives. Papers of Zvi Griliches, Box 129, Folder: “Correspondence, 1954-1959.”

Image Source:  Zvi Griliches from the University of Chicago Photographic Archive, apf1-06565, Special Collections Research Center, University of Chicago Library.

Categories
Harvard Syllabus

Harvard. Reading lists for Aggregate Economic Theory, Dorfman. 1962

 

 

“Macro-economics” was explicitly named in the course description for the Harvard undergraduate economics tutorial in 1962-63. However, not a single course included “macroeconomics” in its title. Instead graduate students were treated to “aggregate economic theory”, an early and one might argue more felicitous name than “macroeconomics”.  This post provides the reading list for Robert Dorfman’s aggregate economic theory course. During the second term of 1958-1959 the same course content was taught by Dorfman as “Economics 241. Money and Banking”.

_________________________

Course Announcement

Economics 241. Aggregate Economic Theory

Half course (fall term). M., W., (F.), at 12. Professor Dorfman.

Source: Harvard University, Faculty of Arts and Sciences. Courses of Instruction for Harvard and Radcliffe, 1962-1963, p. 106.

_________________________

Fall, 1962

HARVARD UNIVERSITY
Department of Economics

Economics 241
READING LIST NO. 1

TEXTS

J. M. Keynes. The General Theory of Employment, Interest and Money.

American Economic Association, Readings in Business Cycle Theory.

Also recommended:

Alvin H. Hansen, A Guide to Keynes

Introductory Material

A. P. Lerner, “The General Theory (1),” S.E. Harris, ed., The New Economics, Ch. 11.

L. Tarshis, “An Exposition of Keynesian Economics,” R.V. Clemence, ed., Readings in Economic Analysis, Vol. I, pp. 197-208.

Gardner Ackley, Macroeconomic Theory, Chs. II, III, IV.

U.S. Department of Commerce, National Income, 1954 Edition (Supplement to the Survey of Current Business), pp. 27-60 and skim the rest.

T.C. Schelling, “National Income, 1954 Edition,” Rev. of Econ. and Stat., XXXVII, 321-335 (November 1955).

Consumption

J.M. Keynes, General Theory. Book III.

Robert Ferber, “Research on Household Behavior,” Amer. Econ. Rev., LII, 19-63 (March 1962) .

Irwin Friend, Individuals’ Saving, Ch. 8

J.S Duesenberry, Income, Saving and the Theory of Consumer Behavior, Ch. 3.

M. Friedman, A Theory of the Consumption Function, Ch. 9, at least.

G. Haberler, “My. Keynes’ Theory of the Multiplier,” Readings in Business Cycle Theory, Ch. 9.

Fritz Machlup, “Period Analysis and Multiplier Theory,” Readings in Business Cycle Theory, ch. 10.

Investment

J.M. Keynes, General Theory, Chs. 11, 12, 16.

I. Fisher, Theory of Interest, Chs. 5-11.

A. A. Alchian, “The Rate of Interest, Fisher’s Rate of Return over Costs and Keynes’ Internal Rate of Return,” Amer. Econ. Rev., X, 938-943 (December 1955).

J.R. Meyer and E. Kuh, The Investment Decision. Chs. 2, 12.

J.S.  Duesenberry, Business Cycles and Economic Growth, Chs. 3, 5.

F. Modigliani and M.H. Miller, “The Cost of Capital, Corporation Finance and the Theory of Investment,” Amer. Econ. Rev. XLVIII, 261-297 (June 1958).

J.M. Clark, “Business Acceleration and the Law of Demand,” Readings in Business Cycle Theory, Ch. 11.

 

 

Fall, 1962

HARVARD UNIVERSITY
Department of Economics

Economics 241
READING LIST NO. 2

Interest Theory

J. M. Keynes, General Theory. Chs. 13, 14, 15, 17.

G.L.S. Shackle, “Recent Theories Concerning the Nature and Role of Interest,” Economic Journal, 71 (June 1961), 209-254.

A.P. Lerner in S.E. Harris, ed, The New Economics, Chs. 45, 46.

B. Ohlin, “Some Notes on the Stockholm Theory of Saving and Investment,” Readings in Business Cycle Theory, Ch. 5

F. A. Lutz, “The Outcome of the Saving-Investment Discussion,” ibid., Ch. 6.

W. Fellner and H.M. Somers, “Alternative Monetary Approaches to Interest Theory,” Rev. of Ec. And Stat., Feb, 1941.

T. Wilson and P.S.W. Andrews, eds., Oxford Studies in the Price Mechanism, Ch. 1.

R. W. Clower, “Productivity, Thrift, and the Rate of Interest,” Economic Journal, March 1954.

Monetary Theory

Irving Fisher, The Purchasing Power of Money, Chs. 2, 3, 8.

Alfred Marshall, “Minutes of Evidence before the Royal Commission on the Values of Gold and Silver,” Questions 9629-9664 (pp. 34-46), Question 9686 (pp. 51-52).

J.M. Keynes, A Tract on Monetary Reform, pp. 74-87.

A.C. Pigou, “The Value of Money,” in F. A, Lutz and L.W. Mints, eds., Readings in Monetary Theory, Ch. 10

W.F. Crick, “The Genesis of Bank Deposits,” ibid., Ch. 4.

H.S. Ellis, “Some Fundamentals in the Theory of Velocity,” ibid., Ch. 7.

Milton Friedman, “The Quantity Theory of Money—A Restatement,” in M. Friedman, ed., Studies in the Quantity Theory of Money, pp. 3-21.

H. Johnson, “Monetary Theory and Policy,” Am. Ec. Rev., June 1962.

W.J. Baumol, “The Transactions Demand for Cash, Quarterly Journ. of Econ., November 1952.

 

 

Fall, 1962

HARVARD UNIVERSITY
Department of Economics

Economics 241
READING LIST NO. 3

Synthesis of Aggregative Economics

J.M. Keynes, General Theory: Chs. 18, 19, 21.

Franco Modigliani, “Liquidity Preference and the Theory of Interest and Money” in Readings in. Monetary Theory, Ch. 11.

J.R. Hicks, “Mr. Keynes and the ‘Classics’” in Readings in the Theory of Income Distribution, Ch. 24.

A.C. Pigou, “The Classical Stationary State,” Economic Journal, December 1943.

Don Patinkin, “Price Flexibility and Full Employment,” American Economic Review, September 1948.

P.A. Samuelson, “The Simple Mathematics of Income Determination,” in Income, Employment and Public Policy (New York: 1948), 133-155.

D.B. Suite, “Forecasting and Analysis with an Econometric Model.” American Economic Review, March 1962.

Marc Nerlove, “A Quarterly Econometric Model for the United Kingdom,” American Economic Review, March 1962.

Aggregative Models of Economic Growth

R.P. Harrod, Towards a Dynamic Economies, Lecture 3.

E.D. Domar, Essays in the Theory of Economic Growth, Chs. 3-5.

Robert Solow, “A Contribution to the Theory of Economic Growth,” Quarterly Journal of Economics, February 1956.

W.J. Baumol, Economic Dynamics, Ch. 4.

READING PERIOD ASSIGNMENT.

J. M. Keyes, The General Theory of Employment, Interest, and Money, entire.

J.G. Gurley and E.S. Shaw, Money in a Theory of Finance.

Source: Harvard University Archives. Syllabi, course outlines and reading lists in Economics, 1895-2003, Box 8, Folder “Economics, 1962-1963 (1 of 2)”.

Categories
Exam Questions Harvard

Harvard. Mid-year and Year-End Final Exams in Economics and Social Ethics, 1894-1895

 

 

With this post Economics in the Rear-view Mirror adds yet another annual collection of final examination questions for the economics courses offered at Harvard together with the questions from Professor Peabody’s “Ethics of the Social Questions” that covered issues such as poverty, labor relations, and socialism (as opposed to doctrines of individualism). In 1894-95 Frank Taussig was on sabbatical leave in Italy which accounts for his whereabouts that academic year.  Today I learned that “doctrine” was understood as a synonym for “theory” during the gay nineties, see Economics 2 (Economic Theory from Adam Smith to the present time) below.

Exams for one course taught were not included in the published collection of exams. It was Edward Cummings course Economics 14 (Philosophy and Political Economy.—Utopian Literature from Plato’s Republic to the present time). Exams for Economics 14 given in other years have been transcribed and posted.

__________________

1894-95.
PHILOSOPHY 5.

Course Title, Staffing, and Enrollment

[PHILOSOPHY] 5. Professor [Francis G.] Peabody. — The Ethics of the Social Questions. — The questions of Charity, the Family, Temperance, and the various phases of the Labor Question, as problems of practical Ethics. — Lectures, essays, and practical observations. 2 hours.

Total 84: 1 Graduate, 40 Seniors, 15 Juniors, 3 Sophomores, 25 Others.

Source: Harvard University. Report of the President of Harvard College, 1894-1895, p. 59.

 

PHILOSOPHY 5.
THE ETHICS OF THE SOCIAL QUESTIONS
Mid-Year Examination. 1895.

[Omit one question.]

  1. The Ethical Idealism of Plato, of Aristotle, and of Kant, compared with the modern doctrine of duty.
  2. Professor Sumner’s doctrine of the Social Fulcrum vs. the philosophy of scientific charity.
  3. Indicate, very briefly, the place in the History of Philanthropy of:

Frédéric Le Play,
Dorothea Dix,
Pastor von Bodelschwingh,
Charles L. Brace,
Samuel G. Howe.

  1. The Elberfeld System — its organization, officials, relation tomunicipal government, and practical working.
  2. The Liverpool System of Collection.
  3. Mr. Charles Booth’s eight classes of East London,— their definition, dimensions, traits, and proportion. (Labor and Life of the People, I. pp. 37-62.) Mr. Booth’s view of the children of Class E (p. 160).
  4. Compare Mr. Booth’s method and results in East London with his method and results in all London.
  5. Compare the principle as to direct relief of the London Charity Organization Society with that of the Boston Associated Charities. (Loch, Charity Organization, pp. 59, 82.) Which is the sounder principle? Why?
  6. The Belgian Labor Colonies,— their scope and method of classification. Compare their aims with those of the colonies of Holland and Germany.
  7. The Christian doctrine of the Social Order — its principles and its peril.

Source: Harvard University Archives. Harvard University, Mid-year examinations, 1852-1943. Box 3. Bound volume Examination Papers, Mid-Year 1894-95.

 

PHILOSOPHY 5.
THE ETHICS OF THE SOCIAL QUESTIONS.
Year-end Examination. 1895.

  1. Explain the theory of ethics which makes the basis of this course of study; and the way in which this theory is practically illustrated by phases of the modern labor question.
  2. In what respect do the social ideals of Carlyle and Ruskin seem identical, and in what respect do they appear to be inconsistent with each other?
  3. The authorship and the significance of the following phrases:

“There is no wealth but Life…. A strange political economy; the only one, nevertheless, that ever was or can be.”

“I am for permanence in all things. Blessed is he that continueth where he is.”

“The gospel of dilettantism.”

“Roots of honour.”

“Ricardo is the parent of Socialism.”

“The value of a thing is independent of opinion and of quantity.”

“The reformation was the work of a monk; the revolution must be the work of a philosopher.”

“The people are the Rock on which the Church of the future must be built.”

  1. The practical programme proposed by Scientific Socialism; the chief advantages claimed for it by its adherents; and the criticisms on it which appear to you most serious. Utilize here your reading of Naquet and The Social Horizon.
  2. Socialism and Religion. The apparent grounds for sympathy and the practical reasons for antagonism. The teachings concerning socialism in the Encyclical of 1891.
  3. The philosophy of history which encourages the Socialist, and the “Opportunist’s” view of this “Law” of social evolution.
  4. The growth of Trades Unionism in Great Britain, and its contribution to moral education.
  5. Federalism and Individualism in English Coöperation. The issue involved, and the advantages of each scheme of expansion.
  6. Compare the characteristics of the forms of Liquor Legislation in force in Massachusetts and in Pennsylvania. (Fanshawe, XI, XII.) How are licenses granted under the Brooks Law? What is the function of probation-officers in Massachusetts?
  7. How far do physiological considerations go to determine one’s duty as to drink?

Source: Harvard University Archives. Harvard University, Examination Papers, 1873-1915. Box 4. Bound volume: Examination Papers, 1893-95. “Papers Set for Final Examinations in Philosophy, History, Government and Law, Economics Fine Arts, Architecture, and Music in Harvard College, June, 1895,” pp. 6-7.

__________________

1894-95.
ECONOMICS 1.

Course Title, Staffing, and Enrollment

[ECONOMICS] 1. Professor [William] Ashley, Asst. Professor [Edward] Cummings, Dr. [John] Cummings, and Mr. [Frederick Redman] Clow. — Outlines of Economics. — Mill’s Principles of Political Economy. — Lectures on Economic Development, Distribution, Social Questions, and Financial Legislation. 3 hours.

Total 277: 2 Graduates, 39 Seniors, 18 Juniors, 159 Sophomores, 9 Freshmen, 50 Others.

Source: Harvard University. Report of the President of Harvard College, 1894-1895, p. 62.

 

ECONOMICS 1.
Mid-Year Examination. 1895.

[Arrange your answers strictly in the order of the question. One question may be omitted.]

  1. “All members of the community are not laborers, but all are consumers, and consume either unproductively or productively.” Explain and illustrate by examples. Suppose everybody resolved to consume “productively” only, what would be the result?
  2. “The distinction, then, between capital and not-capital, does not lie in the kind of commodities, but in the mind of the capitalist — in his will to employ them for one purpose rather than another.” Discuss this statement, using the following illustrations:—

Bread.
A knitting machine.
A steam engine.
A carriage.

  1. Where does true economic rent appear in the following cases:—

(a) The cultivation of a farm by its owner.
(b) The rental of a farm under a long lease by a tenant who has made permanent improvements on the land.

  1. What is the effect on values of a general fall of profits? Of a general fall of wages?
  2. What is the effect on rents of (1) an improvement in the methods of agriculture, (2) an improvement in transportation?
  3. “The price of land, mines, and all other fixed sources of income, depends on the rate of interest.” Explain.
  4. According to Mill, “Every addition to capital gives to labor either additional employment, or additional remuneration.” Why? What is the effect of an increase of labor-saving machinery on employment and on remuneration? Illustrate carefully.
  5. “Money cannot in itself perform any part of the office of capital, since it can afford no assistance to production.” Do you agree or disagree? Why? Is money capital? Is credit money? Is credit capital?
  6. What does Mill mean by “stationary state”? And what changes would bring about a progressive state?
  7. What would be the effect on prices of (1) adding to a gold and silver currency a small issue of inconvertible paper money, (2) the discovery of very rich gold fields?
  8. What do you understand by the Domestic system? By Competition? By Labor?

Source: Harvard University Archives. Harvard University, Mid-year examinations, 1852-1943. Box 3. Bound volume Examination Papers, Mid-Year 1894-95.

 

ECONOMICS 1.
Year-end Examination. 1895.

(Arrange your answers strictly in the order of the question. Omit three of the even numbers: answer all others.)

  1. “We must suppose the entire savings of the community to be annually invested in really productive employment within the country itself; and no new channels opened by industrial inventions, or by more extensive substitution of the best known processes for inferior ones.” How would profits be affected supposing population (a) to remain stationary; (b) to increase in proportion to the increase in capital?
  2. The operations, therefore, of speculative dealers, are useful to the public whenever profitable to themselves; and although they are sometimes injurious to the public, by heightening the fluctuations which their more usual office is to alleviate, yet, whenever this happens the speculators are the greatest losers. Explain Mill’s reasoning.
  3. Mill says of the stationary state, “I am inclined to believe that it would be, on the whole, a very considerable improvement on our present condition.” Why? Explain carefully.
  4. Is there a necessary hostility of interests between consumers organized in co-operative associations and producers organized in trade unions?
  5. Describe the different results obtained in co-operation by distributing profits in the form of dividend (a) on capital, (b) on labor (in proportion to wages), (c) on purchases. Illustrate by the experience of co-operation in France and England.
  6. How do you distinguish between what Mill calls the necessary and the optional functions of government?
  7. “We have had an example of a tax on exports, that is, on foreigners, falling in part on ourselves. We shall therefore not be surprised if we find a tax on imports, that is, on ourselves, partly falling on foreigners.” Explain carefully each case, tracing the possible effects upon prices and international trade of taxes (a) upon exports; (b) upon imports.
  8. “Equality of taxation, therefore, as a maxim of polities, means equality of sacrifice.” Apply this maxim to a tax on incomes.
  9. Suppose a tax of a fixed sum per bushel to be laid upon corn; what would be the effect (a) upon prices; (b) upon population; (c) upon profits; (d) upon rents?
    How would the results differ if instead of a fixed sum per bushel the tax were…

(i) …a fixed proportion of the produce;
(ii) …proportioned to the rent of the land;
(iii) …a fixed sum of so much per cultivated acre? Explain carefully each case.

  1. Describe the kinds of currency used in the United States, indicating briefly the conditions of issue in each case.
  2. Explain the causes and effects of (a) combined reserves, (b) a suspension of the Bank Charter Act in England.
  3. What are the provisions of the law in regard to the issue of bank notes at the present day in England? In Germany?

Source: Harvard University Archives. Harvard University, Examination Papers, 1873-1915. Box 4. Bound volume: Examination Papers, 1893-95. “Papers Set for Final Examinations in Philosophy, History, Government and Law, Economics Fine Arts, Architecture, and Music in Harvard College, June, 1895,” pp. 33-34.

__________________

1894-95.
ECONOMICS 2.

Course Title, Staffing, and Enrollment

[ECONOMICS] 2. Professors Ashley and [Silus Marcus] Macvane. — Economic Theory from Adam Smith to the present time. — Selections from Adam Smith and Ricardo. — Modern Writers. — Lectures. 3 hours.

Total 34: 9 Graduates, 14 Seniors, 6 Juniors, 1 Sophomores, 4 Others.

Source: Harvard University. Report of the President of Harvard College, 1894-1895, p. 62.

 

ECONOMICS 2.
Mid-Year Examination. 1895.

N.B.—Not more than seven questions must be attempted.

  1. “The study which lately in England has been called Political Economy is, in reality, nothing more than the investigation of some accidental phenomena of modern commercial operations, nor has it been true in its investigation even of these. It has no connection whatever with political economy, as understood and treated of by the great thinkers of past ages; and as long as its unscholarly and undefined statements are allowed to pass under the same name, every word written on the subject by those thinkers—and chiefly the words of Plato, Xenophon, Cicero, and Bacon—must be nearly useless to mankind” (Ruskin, Munera Pulveris). Consider some or all of these assertions.
  2. Give a brief account of the Physiocrat doctrine, and state to what extent it was “corrected” by Adam Smith.
  3. Explain the origin and content of Adam Smith’s conception of “Nature.”
  4. “A diamond has scarcely any value in use.” Consider this statement in its relation to the discussion since Adam Smith’s time of the doctrine of Value.
  5. How does the doctrine of Rent expounded by Adam Smith agree with, and differ from, that of Ricardo?
  6. Compare Adam Smith’s “natural rate of wages” with Ricardo’s “natural price of labour.”
  7. “Population tends to outstrip the means of subsistence.” Distinguish the various meanings assignable to this phrase, and indicate which was meant by Malthus.
  8. What does Adam Smith understand by “Capital”? Compare his conception with that of John Stuart Mill.
  9. Present a critical estimate—based upon your own study—of one of the following:

1. Ingram, History of Political Economy.
2. Price, Political Economy in England.
3. Cossa, Introduction to the Study of Political Economy.

Source: Harvard University Archives. Harvard University, Mid-year examinations, 1852-1943. Box 3. Bound volume Examination Papers, Mid-Year 1894-95.

 

ECONOMICS 2.
Year-end Examination, 1895

Answer at least four, but not more than six, of the following questions:

  1. What is the economic source of Interest? Examine the proposition that “interest is the price paid for the use of capital.”
  2. State briefly your conclusions as to the law of general wages.
  3. Apply the Austrian theory of wages to the following case:
    Number of laborers 1,000,000; total subsistence fund $600,000,000; scale of increase of productiveness of labor as the “productive period” is lengthened from one year to seven years: $350, $450, $530, $580, $620, $650, and $670.
  4. How, in your opinion, are the profits of employers determined? What is your conclusion as to the function, in distribution, of the so-called “no profits employers.”
  5. Discuss the following passages:
    “This National Dividend is at once the aggregate Net product of, and the sole source of payment for, all the agents of production within the country: it is divided up into Earnings of labour, Interest of capital, and lastly the Producer’s Surplus, or Rent, of land and of other differential advantages for production. It constitutes the whole of them and the whole of it is distributed among them.”
    “The proposal to put rent aside while we are considering how earnings and interest are determined, has been found to suggest that rent is determined first and then takes part in determining earnings and interest; and this is, of course, the opposite of what really occurs.”
  6. It has been said that Mill expresses his meaning badly when he said that demand for commodities is not a demand for labor. Does the proposition seem to you to need revision!
  7. Does increase of saving tend to make the supply of goods outrun the demand for goods?
  8. Examine the doctrine that the exchange value of commodities is determined by marginal utility.
  9. Past and present relations between gold and silver.

Source: Harvard University Archives. Harvard University, Examination Papers, 1873-1915. Box 4. Bound volume: Examination Papers, 1893-95. “Papers Set for Final Examinations in Philosophy, History, Government and Law, Economics Fine Arts, Architecture, and Music in Harvard College, June, 1895,” p. 35.

__________________

1894-95.
ECONOMICS 3.

Course Title, Staffing, and Enrollment

[ECONOMICS] 3. Asst. Professor Cummings. — The Principles of Sociology. — Development of the Modern State, and of its Social Functions. 2 hours.

Total 52: 10 Graduates, 30 Seniors, 4 Juniors, 3 Sophomores, 5 Others.

Source: Harvard University. Report of the President of Harvard College, 1894-1895, p. 62.

 

ECONOMICS 3.
Mid-Year Examination. 1895.

Answer the questions in the order in which they stand. Omit three questions.

  1. State accurately the reading you have done in this course to date.
  2. “But now let us drop the alleged parallelism between individual organizations and social organizations. I have used the analogies elaborated but as a scaffolding to help in building up a coherent body of sociological inductions. Let us take away the scaffolding: the inductions will stand by themselves.” What are these inductions?
  3. “The family relinquishes one provisional and temporary function after another; its only purpose being to fill gaps in social offices, it made way for independent institutions … as soon as these institutions arose.” Explain and illustrate. How far would Spencer assent to this doctrine?
  4. “Most anthropologists who have written on prehistoric customs believe, indeed, that man lived originally in a state of promiscuity or ‘communal marriage’; but we have found this hypothesis is essentially unscientific.” Discuss the evidence.
  5. “The status of children, in common with that of women, rises in proportion as the compulsory coöperation characterizing militant societies, becomes qualified by the voluntary coöperation characterising industrial societies.” Why? Trace the rise, and illustrate.
  6. “These three distinct states of mind, all of which, in point of fact, are admitted to exist together at the present time, and perhaps to have always done so to a greater or less extent, Comte declares to have undergone a regular progressive movement in the history of society. There have been three successive epochs, during which these philosophic principles, each in its turn, preponderated over both the others and controlled the current of human events.” Explain.
  7. “So that as law differentiates from personal commands, and as morality differentiates from religious injunctions, so politeness differentiates from ceremonial observance. To which I may add, so does rational usage differentiate from fashion.” Explain and illustrate.
  8. How does Spencer account for the diverse types of political organization; and what influences determine the order in which they arise? Illustrate.
  9. “From the Evolution-standpoint we are thus enabled to discern the relative beneficence of institutions which, considered absolutely, are not beneficent; and are taught to approve as temporary that which, as permanent, we abhor.” Explain and illustrate. Does our idea of progress then include all social changes?
  10. “In all ways, then, we are shown that with this relative decrease of militancy and relative increase of industrialism, there has been a change from a social order in which individuals exist for the State, to a social order in which the State exists for individuals?” Explain and illustrate.
  11. According to Spencer, what are likely to be the future forms of political organization and action in societies that are favorably circumstanced for carrying social evolution to its highest stage?
  12. “At bottom this is a physical explanation, and Spencerian sociology in general, whether formulated by Mr. Spencer or by other writers under the influence of his thought, is essentially a physical philosophy of society, notwithstanding its liberal use of biological and psychological data.” Do you agree or disagree? Why?

Source: Harvard University Archives. Harvard University, Mid-year examinations, 1852-1943. Box 3. Bound volume Examination Papers, Mid-Year 1894-95.

 

ECONOMICS 3.
Year-end Examination

[Answer the questions in the order in which they stand. Omit one question.]

  1. State accurately the reading you have done in this course to date.
  2. What has been the function of religion in social evolution? (Compare Spencer and Kidd.) Do you find reasons for thinking society will become more religious?
  3. “The only conclusion to which we are brought by this prolonged examination of authorities is that community of land has not yet been historically proved.” Discuss the evidence.
  4. “And as of old, Society and State tend to coincide, political questions to become identical with social questions.” Discuss the historical changes and tendencies in question. Distinguish carefully between Society, the State, the Government, the Nation.
  5. “It is becoming clear that, when people speak of natural rights of liberty, property, etc., they really mean, not rights which once existed and have been lost, but rights which they believe ought to exist, and which would be produced by a condition of society and an ordering of the State such as they think desirable.” Explain. How far do changes in the theory and practice of penal legislation substantiate this view?
  6. “The gulf between the state of society towards which it is the tendency of the process of evolution now in progress to carry us, and socialism, is wide and deep.”
    “The Individualism of the past is buried, and the immediate future is unmistakably with a progressive Socialism, the full extent of which no man can get see.” Discuss carefully the facts and theories upon which these opposing views are based.
  7. “The philanthropic and experimental forms of socialism, which played a conspicuous rôle before 1848, perished then in the wreck of the Revolution, and have never risen to life again.” What were the characteristics of these earlier forms; and what was their relation to the movements which preceded them and followed them?

Source: Harvard University Archives. Harvard University, Examination Papers, 1873-1915. Box 4. Bound volume: Examination Papers, 1893-95. “Papers Set for Final Examinations in Philosophy, History, Government and Law, Economics Fine Arts, Architecture, and Music in Harvard College, June, 1895,” pp. 35-36.

__________________

1894-95.
ECONOMICS 5.

Course Title, Staffing, and Enrollment

[ECONOMICS] 52. Mr. George Ole Virtue. — Railway Transportation. — Lectures and written work. 3 hours. 2d half-year.

Total 21: 2 Graduates, 10 Seniors, 6 Juniors, 1 Sophomores, 2 Others.

Source: Harvard University. Report of the President of Harvard College, 1894-1895, p. 62.

 

ECONOMICS 5.
Year-end Examination. 1895.

  1. Sketch the railroad history of France.
  2. “The [Reilly] bill now before Congress proposes to extend the debt for another fifty years and a grand opportunity will thus be let slip for trying, under the most favorable circumstances, an experiment whose possibilities no man can measure.”
  3. What legislation can you suggest for improving the relations between the different classes of owners of railway capital? For the protection of the interests of investors in railway capital generally?
  4. State briefly the significance in railway history of the following cases: Munn v. Illinois; Wabash, etc. Ry. Co. v. Illinois; Ames v. U. P. Ry. Co.; Budd v. New York; In re Louisville & Nashville; The Denaby Main Colliery Case.
  5. Choose one:

(a) The bearing upon the making of rates, of the “cost of service”; “value of service”; “charging what the traffic will bear”; “joint cost.”
(b) “Group rates,” “equal mileage rates,” “the blanket rate,” “the postage rate,” “Wagen-raum tarif,” “differentials.”
(c) A “reasonable rates.”

  1. Recount the experience which has led the Interstate Commerce Commission to recomment an amendment to the Act to Regulate Commerce: (a) Construing the meaning of “the word ‘line’ when used in the act to be a physical line and not a business arrangement”; (b) relieving “shippers and individuals not connected with railway employment from liability to fine and imprisonment under Section 10,” with certain exceptions.
  2. What would be the probable effect of giving the Commission power to prescribe minimum as well as maximum rates? Would it obviate the necessity now claimed for pooling?
  3. “When the first bill to regulate commerce was passed the great and powerful wedge of State socialism, so far as government control of railroads is concerned, was driven one-quarter of its length into the timber of conservative government. … The pending bill, [the pooling which passed the House at the last session is referred to] the moment it becomes a law, will drive the wedge three-quarters of its length into the timber.”
    Give your reasons for agreeing or disagreeing with each of the above statements.
  4. What conclusions on the question of public management can you draw from the experience of the states in the internal improvement movement?
  5. Why is it peculiarly true in railway business that “competition must end in combination”?
  6. The success of the State Railroad Commissions and suggestions for increasing their efficiency.

Source: Harvard University Archives. Harvard University, Examination Papers, 1873-1915. Box 4. Bound volume: Examination Papers, 1893-95. “Papers Set for Final Examinations in Philosophy, History, Government and Law, Economics Fine Arts, Architecture, and Music in Harvard College, June, 1895,” pp. 36-37.

__________________

1894-95.
ECONOMICS 71.

Course Title, Staffing, and Enrollment

[ECONOMICS] 71. Professor [Charles F.] Dunbar. — The Theory and Methods of Taxation, with special reference to local taxation in the United States. 3 hours. 1st half-year.

Total 28: 6 Graduates, 11 Seniors, 9 Juniors, 2 Sophomores.

Source: Harvard University. Report of the President of Harvard College, 1894-1895, p. 62.

 

ECONOMICS 71.
Mid-year Examination. 1895.

  1. What is the “Benefit Theory” of taxation? What is the “Faculty Theory”? Define “Faculty” as used in this expression.
  2. What are the leading points of difference between the English, Prussian and American income tax systems?
  3. What reasons are there for having income tax levied by national authority rather than local? To what extent, if at all, do these reasons apply also to a general property tax?
  4. In levying a general property tax, should the debts of the taxpayer be deducted from the property held by him?
  5. By what reasoning is it maintained that,—
    “When the local real estate tax is levied according to rental value and assessed in the first instance on the occupier, as is the case in England, the main burden of the tax will rest ultimately on the occupier, not the owner of the premises.”
    Will the same reasoning apply to the income tax on rent, assessed under Schedule A., and collected from the occupier?
  6. What are the leading points of difference between the German method of taxing distilled liquors and the method practised in England and the United States?
  7. The theories of Canard, Thiers and Stein are,—
    “That every tax is shifted on everybody — that every consumer will again shift the tax on a third party, and that this third party who is again a consumer will shift it to someone else — and so ad infinitum. And since everyone is a consumer, everyone will bear a portion of the taxes that everybody else pays.”
    Professor Seligman’s comment is that “the error of this doctrine lies in the failure to distinguish between productive and unproductive consumption.” Is this answer complete? If not, wherein does it fail?
  8. In a statement of the circumstances under which a tax may or may not be capitalized, it is said,—
    “The principle would not apply to special taxes on property or profits if the capital value of this class of commodities should for any other reason fluctuate in price. For example, if a special tax were levied on government securities it might nevertheless happen that if some reason confidence in government bonds, as over against general securities, might decrease to such an extent as to counterbalance the decreased returns from the investment. In such a case there would be no capitalization of the tax.”
    What criticism have you to make on this reasoning?
  9. Can the theory of progressive taxation be satisfied by a gradually decreasing rate of progression [“degressively progressive taxation”]. or does it require a rate which shall cut off all income or accumulation above a certain level?
  10. What practical difficulties does the taxation of real estate offer in shaping a system of progressive taxation?

Source: Harvard University Archives. Harvard University, Examination Papers, 1873-1915. Box 4. Bound volume: Examination Papers, 1893-95. “Papers Set for Final Examinations in Philosophy, History, Government and Law, Economics Fine Arts, Architecture, and Music in Harvard College, June, 1895,” pp. 37-38.

__________________

1894-95.
ECONOMICS 72.

Course Title, Staffing, and Enrollment

[ECONOMICS] 72. Professor Dunbar. — Financial Administration and Public Debts. 3 hours. 2d half-year.

Total 28: 7 Graduates, 11 Seniors, 9 Juniors, 1 Sophomores.

Source: Harvard University. Report of the President of Harvard College, 1894-1895, p. 62.

 

ECONOMICS 72.
Year-end Examination

[Spend an hour on A, and the remainder of the time on B.]

A.

  1. Give an account of the management of the English debt in the decade 1880-90.
  2. Do “sound rules of finance” demand that the principal of the debt or the rate of interest shall be determined by the government? that securities shall never be issued below par? that a government shall not buy in its securities at a premium?

B.

  1. How far, if at all, is the government justified in pledging itself to any fixed policy of debt payment?
    How may the policy of conversation conflict with the policy of debt payment?
  2. Give an account of the United States refunding operations in the decade 1865-75.
  3. Discuss the respective powers of the Secretary of the Treasury of the United States, the Chancellor of the Exchequer in Great Britain, the minister of finance in France.
    In each case where does the responsibility for the financial policy of the government rest?
  4. Give an account of the creation of Pit’s sinking fund and of the successive modifications made in the sinking fund provisions down to 1803.
  5. Discuss the various methods of placing government securities in the market, and the conditions of contract which make one form of security more attractive to buyers than another.
  6. The United States 4 per cent. 30-year bonds are quoted at about 123¼; how is the present worth of these securities determined?
    What determines the present worth of a terminable annuity? of a perpetual annuity? of a life annuity?
  7. Discuss the manner of making up the estimates of public income and expenditure in Great Britain and in France; the manner of providing for any deficits which may occur in any department during the year; the manner of providing for carrying on the government where the enactment of the budget is delayed until after the beginning of the year; and the disposal of balances unexpended at the end of the year.

Source: Harvard University Archives. Harvard University, Examination Papers, 1873-1915. Box 4. Bound volume: Examination Papers, 1893-95. “Papers Set for Final Examinations in Philosophy, History, Government and Law, Economics Fine Arts, Architecture, and Music in Harvard College, June, 1895,” p. 39.

__________________

1894-95.
ECONOMICS 8.

Course Title, Staffing, and Enrollment

[ECONOMICS] 81. Professor Dunbar. — History of Financial Legislation in the United States. 2 hours.

Total 52: 5 Graduates, 22 Seniors, 22 Juniors, 3 Others.

Source: Harvard University. Report of the President of Harvard College, 1894-1895, p. 62.

 

ECONOMICS 8.
Mid-Year Examination. 1895.

  1. Hamilton is sometimes said to have favored the policy of perpetual debt, and Gallatin, on the other hand, to have established the policy of debt-payment. How far are these statements confirmed by the measures of Hamilton and Gallatin respectively?
  2. How far should you say that Hamilton was justified in his expectation (stated in the Report on Public Credit), (1) That the public debt, if properly funded, would answer most of the purposes of money, and (2) that it would increase the amount of capital for use in trade and lower the interest of money?
  3. When were the several classes of obligations in which the revolutionary debt was funded finally paid off?
  4. Was it fortunate or unfortunate that Congress did not adopt Madison’s policy as to a United States Bank in January, 1815? Why?
  5. Give a list, with dates, of the cases in which bills for establishing a United States Bank have been vetoed.
  6. Give as complete a chronology as you can of the events connected with the Bank, from President Jackson’s first attack upon it down to its final failure.
  7. The removal of the deposits is sometimes spoken of as a fatal blow to the United States Bank. What do you gather from your reading as to its importance as regards the business position or credit of the Bank?
  8. What was the Specie Circular of 1836, and what serious financial results did it produce?
  9. What led to the adoption of the National Bank system in 1863?
  10. How would it have eased the financial difficulty in 1861, if the Secretary of the Treasury had made more free use of his authority, under the act of August 5, for suspending some of the provisions of the Independent Treasury act?
  11. The earlier legal-tender acts provided for funding the notes, at the pleasure of the holder, in United States bonds. When and why was this privilege of funding withdrawn? What would probably have been the effect if it had been retained until the close of the war?
  12. What were the steps by which the legal tender issues came to be treated as the practically permanent element in our paper currency and to be fixed in amount?

Source: Harvard University Archives. Harvard University, Mid-year examinations, 1852-1943. Box 3. Bound volume Examination Papers, Mid-Year 1894-95.

__________________

1894-95.
ECONOMICS 9.

Course Title, Staffing, and Enrollment

[ECONOMICS] 9. Asst. Professor Cummings. — The Social and Economic Condition of Workingmen in the United States and in other countries. 3 hours.

Total 79: 3 Graduates, 34 Seniors, 31 Juniors, 5 Sophomores, 6 Others.

Source: Harvard University. Report of the President of Harvard College, 1894-1895, p. 62.

 

ECONOMICS 9.
Mid-Year Examination, 1895.

(Arrange your answers, in the order in which the questions stand. So far as possible illustrate your discussions by a comparison of the experience of different countries.)

  1. State accurately the reading you have done in this course to date.
  2. “The interests of the working classes are identical in all lands governed by capitalist methods of production. The extension of the world’s commerce and production for the world’s markets, make the position of the workman in any one country daily more dependent upon that of the workmen in other countries.” Why? Explain how in the history of trade unions this community of interest among workmen, not only of the same trade and the same country but of different trades and different countries, has actually manifested itself. Illustrate.
  3. Precisely what answer to the “lump of labor” theory is to be drawn from that version of the wage-fund doctrine adopted by Mill, by Walker, by yourself?
  4. How far has the theory and the practice of coöperation offered a complete remedy for the evils of the existing industrial organization? and at precisely what points has the theory and the practice broken down? Illustrate carefully.
  5. “The struggle of the working classes against capitalist exploitation must of necessity be a political struggle.” How far does the history of trade unions and of coöperation show a tendency in this direction? Illustrate carefully.
  6. “But above all things, observe that all types of piece wage, whether single or progressive, and whether individual or collective, possess this most marked superiority over Profit-sharing.” … “At the same time, it is right to remark that there are many cases, in which the method of Profit-sharing surpasses in important respects any form of the ordinary wage-system.” Explain carefully the grounds of the alleged inferiority and superiority in each case.
  7. “Before, therefore, the trade union can realize its policy of ‘collective bargaining,’ it must solve the two-fold problem – how to bind its own constituents, and how to obtain the recognition of employers.” By what methods have trade unions endeavored to solve this problem? Illustrate.
  8. Trace the successive stages of the so-called “industrial revolution” during the last hundred and fifty years.

Source: Harvard University Archives. Harvard University, Mid-year examinations, 1852-1943. Box 3. Bound volume Examination Papers, Mid-Year 1894-95.

 

ECONOMICS 9.
Year-end Examination. 1895.

[Arrange your answers in the order in which the questions stand. So far as possible illustrate your discussions by statistical and descriptive matter showing the relative condition of working people in the United States and in other countries.]

I.

State accurately the reading you have done in this course since the mid-year examinations.

II.

Devote three hours to a careful discussion of the merits and defects of the German system of compulsory insurance, under the following general heads:

  1. An accurate account of the origin, scope, organization, administration of the system in Germany, — stating approximately the numbers insured, the cost of insurance to all parties concerned, the benefits provided, the methods of collection, distribution, etc.;
  2. Difficulties, opposition, and criticisms thus far encountered;
  3. Progress of similar movements towards compulsory insurance in other countries;
  4. Facts bearing upon the adequacy of existing provisions for sickness, accident, old age in England and the United States;
  5. A biographical sketch showing at what age and in what respects the State already interferes to prescribe conditions of employment, education, etc., for operatives reared from childhood to old age in the factory system of Massachusetts: showing also the additional interference which would be involved in the adoption of the German system of compulsory insurance;
  6. Conclusion.

Source: Harvard University Archives. Harvard University, Examination Papers, 1873-1915. Box 4. Bound volume: Examination Papers, 1893-95. “Papers Set for Final Examinations in Philosophy, History, Government and Law, Economics Fine Arts, Architecture, and Music in Harvard College, June, 1895,” p. 40.

__________________

1894-95.
ECONOMICS 10.

Course Title, Staffing, and Enrollment

[ECONOMICS] 10. Professor Ashley. — The Elements of Economic History from the Middle Ages to Modern Times. 2 hours.

Total 61: 9 Graduates, 20 Seniors, 21 Juniors, 10 Sophomores, 1 Other.

Source: Harvard University. Report of the President of Harvard College, 1894-1895, p. 62.

 

ECONOMICS 10.
Mid-Year Examination. 1895.

I. To be first attempted by all.

Translate, and comment on, the following passages:

  1. Quomodo vocatur mansio; quis tenuit eam T. R. E.; quis mod tenet; quot hidae; quot carrucae in dominio; quot hominum; quot villani; quot cotarii; quot servi; quot liberi homines; quot sochemanni.
  2. De virgis operantur ii diebus in ebdomada.
  3. Rex. . . destinavit per regnum quos ad id prudentiores.. . . cognoverat, qui circumeuntes et oculata fide fundos singulos perlustrantes, habita aestimatione victualium quae de hiis solvebantur, redegerunt in summam denariorum.
  4. Interiors plerique frumenta non serunt, sed lacte et carne vivunt, pellibusque sunt vestiti.
  5. Ideo rogamus, sacratissime imperator, subvenias. . . . ademptum sit jus etiam procuratoribus, nedum conductori, adversus colonos ampliandi partes agrarias.
  6. Arva per annos mutant et superest ager.
  7. Ego Eddi episcopus terram quae dicitur Lantocal tres cassatos Heglisco abbati libenter largior.
  8. Rex misit in singulos comitatus quod messores et alii operarii non plus caperent quam capere solebant.
  9. Noveritis nos concessisse omnibus tenentibus nostris . . . . quod omnia praedicta terrae et tenementa de cetero sint libera, et liberae conditionis.

II. Write on two only of the following subjects.

  1. The reasons for believing in the survival in Britain of the Roman agrarian organisation.
  2. A comparison, from the economic point of view, of the open-field system with modern methods of farming.
  3. The condition of the tillers of the soil in England in A.D. 1381 as compared with A.D. 1066.

Source: Harvard University Archives. Harvard University, Mid-year examinations, 1852-1943. Box 3. Bound volume Examination Papers, Mid-Year 1894-95.

 

ECONOMICS 10.
Year-end Examination. 1895.

I.
[To be first attempted by all.]

TRANSLATE, and comment on, the following passages:—

  1. In Kateringes sunt x. hidae ad geldum Regis. Et de istis
  2. hidis tenent xl. villani xl. virgas terrae. … Et omnes isti homines operantur iiibus diebus in ebdomada.
  3. Agriculturae non student; majorque pars eorum victus in lacte caseo carne consistit; neque quisquam agri modum certum aut fines habet proprios; sed magistratus ac principes in annos singulos gentibus cognationibusque hominum qui una coierunt, quantum et quo loco visum est agri attribuunt atque anno post alio transire cogunt.
  4. Nul ne deit rien acheter a revendre en la vile meyme, fors yl serra Gildeyn.
  5. Cives Londoniae debent xl marcas pro Gilda Telaria delenda; ita ut de cetero non suscitetur.
  6. Johannes Hore mortuus est, qui tenuit de domino dimidiam acram terrae cujus heriettum unus vitulus precii iiii d. Et Johanna soror dicti Johannis est proximus heres, quae venit et gersummavit dictam terram tenendam sibi et suis in villenagio ad voluntatem per servicia et consuetudines.

II.
[Write on four only of the following subjects.]

  1. “I contend that from 1563 to 1824, a conspiracy, concocted by the law and carried out by parties interested in its success, was entered into, to cheat the English workman of his wages, to tie him to the soil, to deprive him of hope, and to degrade him into irremediable poverty.” Consider this.
  2. Discuss the question whether the statute of 5 Eliz. c. 4, displays any distinct economic policy.
  3. Explain the causes, nature and consequences of the change in commercial routes in the sixteenth century.
  4. What is meant by a national economy, as contrasted with a town economy? Illustrate from European conditions in the 15th and 16th centuries.
  5. What has been the economic gain to England from immigration?
  6. Mention briefly those respects in which the economic development of England has resembled that of Western Europe, and those respects in which it has been peculiar.

Source: Harvard University Archives. Harvard University, Examination Papers, 1873-1915. Box 4. Bound volume: Examination Papers, 1893-95. “Papers Set for Final Examinations in Philosophy, History, Government and Law, Economics Fine Arts, Architecture, and Music in Harvard College, June, 1895,” pp. 40-41.